You are on page 1of 87

KAPLAN LSAT PREP

LSAT

RELEASED TEST I
EXPLAINED

A Guide to the June, 1991 LSAT

KAPLAN
The answer to the test question.
„1996 Stanley H. Kaplan Educational Center Ltd

All rights reserved. No part of this book may be reproduced in any form, by
photostat, microfilm, xerography or any other means, or incorporated into any
information retrieval system, electronic or mechanical, without the written permission
of Stanley H. Kaplan Educational Center Ltd.
SECTION I:
READING COMPREHENSION

© K A PL A N 1
LSAT PREP __________________________________________________________________ LSAT Test I Explained: Section I

Passage 1—Phillis Wheatley


(Q. 1-8)

Topic and Scope: Phillis Wheatley, specifically Wheatley’s poetry.

Purpose and Main Idea: The author’s purpose is to discuss and evaluate Wheatley’s
poetry. The author’s specific main idea is that Wheatley was an accomplished poet who
rightly deserves to be known as the first African American poet, though her poetry was not
influenced by African traditions and did not contribute to the growth of an African
American literary tradition.

Paragraph Structure: ¶1 states that Wheatley quickly became an accomplished poet, even
though she was not raised in an English-language environment. ¶2 digresses from the topic
in order to discuss the background of African American English.

¶3 reveals the author’s belief that Wheatley’s poetry was not influenced by her African
heritage. ¶4 summarizes the author’s feelings about Wheatley: though she did not
contribute to the growth of an African American literary tradition, she was nevertheless an
accomplished poet who deserves to be recognized as the first African American poet.

The Big Picture:

• Topic and scope are evident early on in the passage, but purpose and main idea are
not. Instead, the passage hits you with a lot of details. It’s not until ¶s 3 and 4 that
you get a strong sense of authorial purpose and main idea. On Test Day, it might be
better to leave a similarly structured passage for later in the section. In general, it’s
best to begin your work on the Reading Comprehension section with a passage
whose purpose and main idea (if there is one) are apparent early in the text.

• Don’t assume that all the key information is contained in the first ¶. In this passage,
for instance, most of the important information emerges toward the end. Always
apply your critical reading skills to the entire passage.

The Questions:

1. (C)
This choice nicely captures the author’s main idea. Although Wheatley was an
accomplished poet, her poetry did not reflect the traditions of her African heritage.

(A) is beyond the scope of the text. This passage is specifically about Wheatley’s poetry. It
doesn’t directly compare her poetry to the work of folk artists.

(B) is a “half right, half wrong” choice. The first part of this choice is okay, but the second
goes awry in suggesting that Wheatley was influenced by African traditions.

(D) and (E) According to the author, Wheatley’s poetry was disconnected from African
traditions and African American literary developments.

2 © K A PL A N
LSAT PREP __________________________________________________________________ LSAT Test I Explained: Section I

• The correct answer to a “global” question must be broad enough to encompass the
contents of the entire passage, yet narrow enough to refer to the specific subject
matter of the passage.

• Often, the wrong answers to global questions are in direct conflict with the author’s
point of view. So when you see a choice that represents the opposite of your pre-
phrase, don’t panic. You’re probably on the right track. Expect to see a few au
contraire answer choices, and you’ll eliminate them with confidence.

2. (E)
According to the passage, Wheatley’s poetry was influenced by the English literary
conventions of her day. Her poetry was neither influenced by nor reflected her ethnic
heritage. A contemporary Italian immigrant poet who is influenced solely by the
conventions of modern American poetry would be following a similar approach to poetry
as Wheatley.

(A), (B), (C), (D) All of these choices go wrong in suggesting that Wheatley’s work was
somehow influenced by or reflected her ethnic heritage.

• In questions that ask you to compare hypothetical situations to a situation described


in the passage, look for the choice that mimics the relationship outlined in the
passage.

3. (D)
Lines 19-24 make precisely this point.

(A) and (B) The passage mentions African American religious music and folk art, but not
the religious music and folk art of New England colonists. New England colonists
themselves, as a matter of fact, aren’t even mentioned in the passage.

(C) According to lines 19-24, African languages affected spoken English, not formal written
English.

(E) The passage doesn’t discuss general “eighteenth-century aesthetic principles,” and
there’s nothing in the passage to suggest that African languages had an impact on them.

• If you’re in doubt about a detail, be sure to check back with the passage, using your
“roadmap” of the passage to locate the relevant text. Don’t answer on a hunch or a
vague recollection.

© K A PL A N 3
LSAT PREP __________________________________________________________________ LSAT Test I Explained: Section I

4. (D)
The phrase “closed system” emerges in the context of the author’s remark that eighteenth-
century English poetry did not incorporate the informal, popular language of its day.
Hence, by a closed system of poetry the author undoubtedly means poetry that “does not
admit the use of street language and casual talk.”

(A) If anything, the author believes that poetry can be written by someone who isn’t raised
knowing its conventions. After all, as the author points out, Wheatley wrote poetry based
on conventions that she learned quickly, after she had reached New England.

(B) and (E) are beyond the scope of the text. The author never discusses poetry’s impact on
spoken language (B). Nor does she discuss why poetry might ultimately be rejected (E).

(C) In lines 35-36, the author says that poetry in a “closed system” is based on imitating
past works.

• Whenever a question asks you to interpret the meaning of a word or phrase, read the
lines around that word or phrase. The key to understanding what the author means
by a word or phrase lies in understanding the context in which it appears.

5. (A)
Line 41 states that the conventions of eighteenth-century English poetry permitted the poet
to express “generalized feelings.”

(B), (D), (E) The conventions of this poetry, according to line 40, did not allow “casual
talk” (D). The ban on casual talk, of course, would also have eliminated “Americanized
English” (B) and “Black speech” (E). Indeed, their absence from Wheatley’s poetry is
confirmed by lines 36-38.

(C) The lack of “themes from folk art” in Wheatley’s poetry is highlighted in lines 42-49.

• This question illustrates the importance of having an awareness of paragraph


structure. If you kept in mind where in the passage the author evaluates Wheatley’s
poetry, you should have gone directly to ¶3 for the answer to this question.

6. (E)
In lines 54-56, the author claims that “Wheatley’s poetry contributed little to the
development of a distinctive African American literary language.” Thus, if scholars were
to “trace themes and expressions in African American poetry back to the poetry of
Wheatley,” the author’s argument concerning Wheatley’s role in the evolution of an
African American literary language would be seriously weakened.

(A) Since the author claims that Wheatley’s poetry conformed to the conventions of
neoclassical poetry, her argument would not be undermined by a finding that Wheatley’s
poetry was admired in England because of its adherence to English literary traditions.

4 © K A PL A N
LSAT PREP __________________________________________________________________ LSAT Test I Explained: Section I

(B) Why would the author’s argument about Wheatley’s poetry be weakened by the
discovery that Wheatley had written a history of her family’s experiences?

(C) Likewise, why would the author’s argument about Wheatley’s poetry be weakened by
the knowledge that other Africans were able to overcome the language barriers that she
overcame?

(D) misrepresents the author’s criticism. The author argues that Wheatley failed to play a
meaningful role in the development of an African American literary language. Whether
Wheatley was an important poet in the American literary tradition is another issue
altogether.

• In questions that ask you to pick the choice that would most strengthen or weaken
an authorial position, read the question stem carefully to be sure which you’re asked
to do. Often, an incorrect choice will do the exact opposite of what you’re asked to
do, and you could fall for this trap choice if you haven’t read the question stem
carefully.

• Don’t bend over backwards for an answer choice. If the relevance of a choice
depends upon making assumptions unsupported by the passage, then that choice is
irrelevant. For example, you could spend precious time on choice (B) attempting to
find a connection between a family history and the evolution of an African
American literary language, but what would be the point?

7. (D)
In lines 30-31, the author expresses dismay that Wheatley did not integrate African
traditions into her poetry in order to create an African American literary language. It is
inferable, therefore, that the author would have praised Wheatley’s work even more had
Wheatley done so.

(A) “Black English” is mentioned in the third ¶ as a potential source that Wheatley could
have used in creating an African American literary language. However, the author never
criticizes Wheatley for failing to influence the way English is spoken.

(B) The author would have praised Phillis Wheatley more if she had invented an African
American literary tradition.

(C) To the extent that the author is critical of Wheatley, it’s because Wheatley so strictly
adhered to the conventions of eighteenth-century English poetry.

(E) The author doesn’t bring up the general issue of “the barriers that written English
literary forms presented to Black authors.”

• This question illustrates the importance of grasping authorial main idea. If you
recognized that the author argues that Wheatley failed to play a role in developing an
African American literary language, settling on choice (D) should have been a snap.

© K A PL A N 5
LSAT PREP __________________________________________________________________ LSAT Test I Explained: Section I

8. (B)
While the author praises Wheatley as an accomplished poet, she also notes that Wheatley
failed to contribute to the African American literary tradition. Thus, the author’s attitude
toward Wheatley’s work is aptly characterized as one of “qualified admiration.”

(A) “Enthusiastic advocacy” suggests that the author is uncritical of Wheatley’s work.

(C) and (D) “Dispassionate impartiality” (C) and “detached ambivalence” (D) suggest that
the author has no real opinion about Wheatley’s work.

(E) “Perfunctory dismissal” suggests that the author condemns Wheatley’s work.

• Watch out for extreme-sounding choices such as (A) and (E). Most LSAT Reading
Comprehension passages are more moderate in tone.

6 © K A PL A N
LSAT PREP __________________________________________________________________ LSAT Test I Explained: Section I

Passage 2—Cell Biology


(Q. 9-16)

Topic and Scope: The relationship between cell biology (cytology) and biochemistry;
specifically, how the two disciplines have affected each other over the last century.

Purpose and Main Idea: The author’s purpose is to describe and evaluate the historical
interaction between cell biology and biochemistry. The author’s main idea is to argue that
the interaction between these two disciplines has lead to progress in both of them.

Paragraph Structure: ¶1 notes that two scientific disciplines concerned with the same
general topic are often in conflict with each other in their early stages of development
because of their different approaches to the topic. One discipline takes a “macro”
approach to the topic, while the other discipline takes a “micro” approach.

¶s 2 and 3 go on to describe a particular historical case of conflict between two related


scientific disciplines: cell biology and biochemistry. ¶4 explains that the claims of both
disciplines have essentially been borne out, leading to a synthesis of the two in the form of
the new discipline of molecular genetics.

¶5 summarizes the author’s main idea: competition between these related scientific
disciplines has been useful, for it has generated important scientific discoveries.

The Big Picture:

• Whenever a passage compares/contrasts entities—as this passage does—make sure


that you can distinguish between (among) the entities. The questions will certainly
test if you can tell them apart.

• Don’t let unfamiliar scientific terms or concepts throw you. As you read, boil a
science passage’s ideas down to their essence. If you simplify things as you go, the
questions will be much less intimidating.

The Questions:

9. (B)
After reviewing (in ¶s 1-3) the process by which related scientific disciplines interact, ¶s 4
and 5 go on to argue that this interaction is often scientifically beneficial.

(A) Neither the author’s general discussion of the interaction between related scientific
disciplines nor his specific example of the interaction between cell biology and
biochemistry suggests that this interaction has negative consequences for the advancement
of science.

(C) The author doesn’t go so far as to contend that the interaction between related
disciplines necessarily leads to a synthesis of their approaches in all cases. After all, he only
looks at one case.

© K A PL A N 7
LSAT PREP __________________________________________________________________ LSAT Test I Explained: Section I

(D) This choice focuses on details in ¶s 2 and 3.

(E) This choice focuses on a detail in ¶5.

• In “global” questions, watch out for choices that focus on one part of the passage
rather than on the entire passage.

10. (E)
This choice nicely paraphrases lines 19-21.

(A) Line 39 mentions “chromosome mapping,” but suggests that this may not have been a
late nineteenth-century development. Moreover, it’s not clear whether cytologists or
biochemists did the mapping.

(B) and (D) According to lines 29-32, late nineteenth-century biochemists, not cytologists,
were concerned with the “chemical nature of protoplasm” (B) and the “role of enzymes in
biological processes” (D).

(C) According to ¶4, biochemists, not cytologists, deal with the “spatial relationship of
molecules within the cell.” Moreover, this issue has been the focus of post-1950
biochemists.

• This question illustrates the importance of checking back with the passage about
details. All of the choices might have looked very tempting if you didn’t consult ¶s
2-4.

11. (A)
If biochemists “stood apart” from the late nineteenth-century debate about the structural
nature of protoplasm (as lines 25-29 say), then it stands to reason that the debate must have
taken place among cytologists.

(B), (C), (E) Again, lines 25-29 say that late nineteenth-century biochemists avoided “the
debate over the structural nature of protoplasm.”

(D) is out for the simple reason that the passage never even mentions “geneticists,” let alone
late nineteenth-century geneticists.

• Sometimes questions can be answered through a process of elimination. If it wasn’t


immediately clear to you that the debate took place among cytologists, you could
have eliminated choices (B)-(E) by recognizing that neither biochemists nor
geneticists were involved in the debate.

8 © K A PL A N
LSAT PREP __________________________________________________________________ LSAT Test I Explained: Section I

12. (B)
This choice nicely paraphrases lines 35-37.

(A) The chemical reactions that occur in cytological preparations (lines 23-24) were an
issue raised by late nineteenth-century biochemists, not late nineteenth-century cytologists.

(C) According to lines 29-31, late nineteenth-century biochemists were interested in the
“nature of protoplasm.”

(D) The only mention of “cell division” occurs in lines 20-21, and this reference has
nothing to do with the criticism of late nineteenth-century biochemists leveled by late
nineteenth-century cytologists.

(E) Late nineteenth-century cytologists criticized the methods, not the knowledge, of late
nineteenth-century biochemists.

• A common “trap” choice is the answer choice that uses the passage’s language but
distorts the passage’s ideas. Such choices are especially common in science
passages, which often contain complex language and unfamiliar terminology.

13. (A)
The first sentence of ¶5 contains the author’s main idea: “[The] interaction between paired
disciplines can have important [scientific] results.” A few lines later, the author quotes a
similar remark by Fruton. The author, in other words, quotes Fruton in order to reinforce
the author’s own conclusion.

(B), (C), (E) Fruton’s remark is general in nature; it doesn’t refer specifically to cytology or
biochemistry.

(D) Like the author, Fruton believes that competition between disciplines generates
progress, not problems.

• Wrong choices are often wrong for the same reason. In this case, note that choices
(B), (C), and (E) all incorrectly claim that Fruton’s remark applies to the specific case
of cytology and biochemistry.

14. (A)
¶2 indicates that the enzyme theory emerged in the late nineteenth century, while ¶4
indicates that the discipline of molecular biology is mainly a post-1950 phenomenon. In
other words, the enzyme theory appeared before molecular biology.

(B) According to ¶2, the initial discovery of cell architecture, like the formulation of the
enzyme theory, occurred in the late nineteenth century.

(C) ¶3 suggests that chromosome mapping may have occurred later than the late
nineteenth century. In any case, it certainly didn’t occur before the late nineteenth century.

© K A PL A N 9
LSAT PREP __________________________________________________________________ LSAT Test I Explained: Section I

(D) The synthesis of ideas between cytologists and biochemists is another post-1950
phenomenon.

(E) Lines 25-32 suggest that the debate about the nature of protoplasm emerged before the
enzyme theory.

• The correct answers to “inference” questions stick closely to the spirit of the text. If
you have to go through an intricate reasoning process in order to justify a choice,
you’ve probably picked an incorrect choice.

15. (B)
Lines 29-31 state that late nineteenth-century biochemists were concerned with the
chemical nature of protoplasm. Hence, it’s inferable that they considered understanding
the nature of protoplasm to be vital to understanding cell processes.

(A) and (D) Late nineteenth-century cytologists, not late nineteenth-century biochemists,
were concerned with the structure of the cell (A) and the physical traits of protoplasm (D).

(C) and (E) are beyond the scope of the passage. There’s no mention of what late nineteenth-
century biochemists may have thought about the “behavioral patterns of organisms” (C).
Nor is there any discussion about whether they thought chemistry was inadequate for the
task of understanding the cell (E).

• It’s important to read the question stem carefully. This particular question asks
about the views of biochemists, not cytologists. If you missed this point because you
read the stem too quickly, you might have fallen for wrong choice (A) or (D).

16. (C)
¶1 presents a general proposition regarding the relationship between two similar scientific
disciplines. The rest of the passage illustrates this proposition through an extended
example: the relationship between cytology and biochemistry as it has developed over the
last century.

(A) What process? What reason for its occurrence?

(B) What set of examples? Only one example—the relationship between cytology and
biochemistry—is explored.

(D) What statement of principles? What rationale?

(E) What problem? What solution?

• Questions with abstractly-worded answer choices aren’t necessarily more difficult


than questions with concretely-worded answer choices. Don’t sacrifice an easy point
out of an irrational fear of abstract language.

10 © K A PL A N
LSAT PREP __________________________________________________________________ LSAT Test I Explained: Section I

Passage 3—Criminal Procedure


(Q. 17-21)

Topic and Scope: The adversarial and inquisitorial systems of criminal procedure;
specifically, the differences between these two forms of criminal procedure.

Purpose and Main Idea: The author’s purpose is to compare/contrast the two systems of
criminal procedure. Although the author largely restricts himself to a factual description
of the two systems, he also argues that the inquisitorial system is superior to the adversarial
system.

Paragraph Structure: ¶1 introduces the two systems of criminal procedure: adversarial


and inquisitorial. ¶s 2 and 3 describe the adversarial and inquisitorial systems,
respectively. ¶s 4 and 5 point out some of the differences between the two systems.

The Big Picture:

• Whenever the passage compares multiple theories or approaches, make sure that
you’re clear about their differences. The questions will test to see whether you’ve
picked up on them.

• In passages that contain a lot of intricate details—passages like this one—it’s a


mistake to try to assimilate the details during a first read through of the text. Instead,
note where in the passage each detail occurs. Doing so will allow you to quickly look
up any detail cited by the questions.

• Don’t give up on harder passages on the assumption that no easy points are
associated with them. Often, a difficult passage will have very accessible questions
(as is the case here).

The Questions:

17. (A)
Lines 47-48 point out that the adversarial system is based on legal rules.

(B) To the extent that reenactments of crimes are associated with either system of criminal
procedure, they are more closely identified with the inquisitorial system, as lines 48-50
suggest.

(C) Lines 44-45 point out that the inquisitorial system is based on a search for the facts.

(D) Personal vengeance is a part of neither the adversarial nor the inquisitorial systems.
These systems, as a matter of fact, replaced the earlier system of private vengeance.

(E) Police testimony has a role to play in both the adversarial and the inquisitorial systems.
It’s not what distinguishes one from the other.

© K A PL A N 11
LSAT PREP __________________________________________________________________ LSAT Test I Explained: Section I

• A strong mental roadmap of the passage makes inference questions much easier.
Knowing where to look is more than half the battle. For example, knowing that the
“crucial factors” of the two approaches are discussed in ¶ 4 makes this question
much less difficult.

18. (E)
Lines 41-43 indicate that the judge is supposed to be an “involved manager” under the
inquisitorial system.

(A) and (C) The word passive in “passive observer” (A) and the word uninvolved in
“uninvolved administrator” (C) run directly counter to the sentiment expressed in lines 41-
43.

(B) and (D) Nothing in the text suggests that the judge under the inquisitorial system is
supposed to be “biased” (B) or “aggressive” (D). He or she is simply supposed to manage
and guide the legal proceedings.

• If you took note of the passage’s structure, it should have been a snap to go back to
lines 41-43 for the answer. If you didn’t note the passage’s structure, you might have
wasted valuable time looking for the lines that contain the answer.

19. (B)
¶1 indicates that “the victim of a crime fashioned a remedy and administered it privately”
under the system of private vengeance. In contrast, under both the adversarial and
inquisitorial systems, society is responsible for taking legal action against a criminal—see
lines 11-14 and lines 30-36 for the adversarial and inquisitorial systems, respectively.

(A) and (C) The defendant is the person accused of committing the crime, and presumably
never had much of an incentive to initiate legal action. Responsibility for initiating legal
action under the system of private vengeance rested with the victim, not the victimizer.

(D) Again, the victim, not the court system, was responsible for taking legal action under
the system of private vengeance. Indeed, the nature of this system suggests that there were
no courts where it held sway.

(E) Again, under the adversarial and inquisitorial systems, society, not judges, are
responsible for taking legal action against criminals.

• Don’t be intimidated by long question stems—they don’t necessarily indicate


difficult questions.

12 © K A PL A N
LSAT PREP __________________________________________________________________ LSAT Test I Explained: Section I

20. (D)
Under the adversarial system, according to lines 17-19, the defendant is responsible for
conducting his own pretrial investigation. In other words, the defendant is responsible for
mounting his own defense. In contrast, lines 30-36 point out that under the inquisitorial
system, the public prosecutor has an important role to play in the defense, which conflicts
with (D).

(A) According to ¶s 3 and 4, both the prosecution and the defense have a duty to search
for the facts of a case under the inquisitorial system, which stands in marked contrast to the
less cooperative adversarial system.

(B) Lines 30-38 make the point that the inquisitorial system “encourages full disclosure of
evidence.”

(C) Lines 41-43 make the point that the judge plays “an active role in the conduct” of a trial
carried out under the inquisitorial system.

(E) Lines 51-55 suggest that the inquisitorial system “favors the innocent.”

• In all/EXCEPT questions, you’re generally asked to look for the choice that isn’t true.

21. (B)
In lines 29-30, the author calls the inquisitorial system “historically superior to the
adversarial system” and in line 51 he trumpets the inquisitorial system’s “thoroughness” in
ascertaining the facts. Thus, choice (B) nicely captures the author’s mood toward the
inquisitorial system.

(A), (D), and (E) mistakenly suggest that the author has qualms about the inquisitorial
system.

(C) The author never claims that the inquisitorial system will replace the adversarial
system; he simply claims that it’s better.

• If you picked up on the author’s positive attitude toward the inquisitorial system as
you read through the passage, you could have immediately eliminated three of the
five choices in this question, making your task of choosing the correct answer much
simpler.

© K A PL A N 13
LSAT PREP __________________________________________________________________ LSAT Test I Explained: Section I

Passage 4—Professions
(Q. 22-28)

Topic and Scope: Professions; specifically, the definition of the term profession.

Purpose and Main Idea: The author’s purpose is to define the term profession in order to
distinguish it from other types of endeavors. The author’s main idea is that a profession is
distinguished from other endeavors by the commitment shown by its practitioners.

Paragraph Structure: ¶1 introduces the issue of distinguishing between a profession and


other endeavors and dips into the history of the term profession. ¶2 and the first part of ¶3
discuss definitions of the term that the author rejects—definitions that differentiate
professions from other endeavors based on learning/knowledge and honor/prestige. The
rest of ¶3 and ¶4 provide the author’s definition of the term profession: a profession is
distinguished from other endeavors by the “devotion” of its members to some noble way
of life.

The Big Picture:

• Notice the structure of this passage: ¶1 introduces a “problem”; ¶s 2 and 3 describe


“solutions” to the problem that the author rejects; and ¶s 3 and 4 describe the
author’s “solution” to the problem. On test day, don’t be surprised if you see a
similarly structured passage.

• Nonscience passages sometimes contain complex prose. As in science passages that


contain complex prose, resist the temptation to understand the passage in detail the
first time through it. Instead, just try to grasp the passage’s purpose and structure.
Only shoot for a finer understanding of the passage if the questions demand it.

The Questions:

22. (E)
This choice nicely captures the sentiments expressed by the author in lines 55-67.

(A) and (C) Others have put forward the notion that professions can be distinguished from
other endeavors based on prestige (A) and knowledge (C). The author himself rejects these
criteria.

(B) In ¶2, the author rejects the notion that “know-how in a particular field” distinguishes
professionals from non-professionals.

(D) The author never links professional status to the pursuit of “political justice.”

• The correct answer to the first question in a set will often be based on information
near the end of the passage. Just another reason to read the entire passage.

14 © K A PL A N
LSAT PREP __________________________________________________________________ LSAT Test I Explained: Section I

23. (D)
The author’s main idea is that professionalism requires a commitment to a noble way of
life. Since the author makes this point by concentrating on the case of the medical
profession, choice (D) is an accurate expression of the author’s main idea.

(A) The etymology of the word profession is a detail that appears in ¶1.

(B) The author never says that it’s wrong to recognize the knowledge and skills of
physicians; he simply thinks that these things aren’t responsible for medicine’s status as a
profession.

(C) The author acknowledges that the work of physicians is under attack, but never claims
that this has resulted from widespread misunderstanding. Besides, this acknowledgment is
just a detail in ¶1.

(E) It’s not physicians who think of themselves as “technicians”; it’s others who think of
them as such. Besides, this, too, is just a detail in ¶1.

• Note that you could have eliminated the four wrong choices simply by recognizing
that they focus on passage details.

• It’s a good idea to read through all of the choices before you endorse any of them.
Choices that focus on details may be tempting at first, so it’s important to give all the
choices a shot.

24. (C)
The entire passage concerns the nature of professions, using the medical profession as an
example.

(A) Efforts to redefine medicine as a trade are mentioned before lines 7-10, but the passage
doesn’t address the question of whether these efforts are likely to succeed.

(B) The author never claims to dislike “governmental regulation” or “consumer advocacy.”
He simply mentions that such things currently pose a challenge to the status of the medical
profession. Moreover, the author mentions these things prior to posing the question in
lines 7-10.

(D) What suggestions? The author never offers suggestions for rallying people to the
defense of physicians, though he personally defends their status.

(E) is beyond the scope of the passage. The author isn’t interested in the origins of words in
general; only the origins of the words trade and profession.

• This question demonstrates the importance of following the flow of the author’s
argument and noting paragraph topics. With a roadmap of the passage, backtracking
to find the role of lines 7-10 is much easier.

© K A PL A N 15
LSAT PREP __________________________________________________________________ LSAT Test I Explained: Section I

25. (D)
The author notes that trades are based on specific knowledge, but never mentions what this
knowledge is.

(A) How society treats physicians is addressed in lines 1-7.

(B) Lines 51-55 draw an analogy between teaching and medicine.

(C) The public nature of a profession is made amply clear in lines 55-60.

(E) Lines 60-63 distinguish a livelihood from a profession.

• Don’t hesitate to use a process-of-elimination strategy if you have to. Choice (D) is a
bit tricky: eliminating the other four choices makes it easier to see that it is indeed
correct.

26. (E)
This sentiment is expressed in ¶s 3 and 4.

(A) What new perspective? The author argues that society should continue to recognize
physicians as professionals.

(B) The author never addresses the issue of whether some professionals have been
demoralized by public opinion.

(C) The author mentions governmental regulation of and litigation against the medical
profession, but doesn’t discuss their effects on the medical profession.

(D) The author never claims that “most professionals have come to be considered
technicians.” He only notes efforts to classify physicians as technicians.

• In “attitude” questions that have long answer choices (i.e., choices that are longer
than one or two words), you must read the whole choice. Don’t pick a choice just
because the first word or two appears to reflect the author’s point of view.

27. (B)
Since the passage ends with the author providing his ethics-based definition of
professionalism, it’s logical that a follow-on ¶ could begin with an objection to that
definition.

(A) The author discusses trades only in ¶1; hence, it’s not likely that he would come back
to this issue in a hypothetical fifth ¶.

(C) The role of the “community” is not something that the author discusses; thus, it’s not
logical to think that he’d suddenly take up this issue in a hypothetical fifth ¶.

16 © K A PL A N
LSAT PREP __________________________________________________________________ LSAT Test I Explained: Section I

(D) In the passage, the author defines the essential nature of the medical profession; so,
why would he suddenly express doubts about this issue?

(E) The passage never refers to the “whole body” or the “meaning of illness”; so, it’s
unlikely that the author would suddenly introduce these ideas in a follow-on ¶.

• Context is all important in a question like this one. To determine what idea would
logically come next, you’ve got to have a sense of what idea the passage closed with.

28. (C)
From the end of ¶1 through the middle of ¶3, as we’ve already noted, the author is
primarily concerned with raising and dismissing alternative definitions of the term
profession.

(A) That “something else” is discussed from line 47 onward.

(B) What efforts? The author dismisses alternative definitions of professionalism, but that’s
not the same thing as saying that he dismisses “efforts” to redefine the term.

(D) Lines 18-42 do contain a discussion of linguistics and history, but only in order to
dismiss alternative definitions of professionalism, not to clarify this term’s meaning.

(E) Distinguishing between trades and professions is part of the author’s larger purpose of
refuting definitions that don’t satisfy him.

• The correct answer to this sort of “mini-global” question must be broad enough to
cover the contents of all of the cited lines, not merely a portion of them.

© K A PL A N 17
SECTION II:
LOGIC GAMES

18 © K A PL A N
LSAT PREP _________________________________________________________________ LSAT Test I Explained: Section II

GAME 1 — Trade Representatives


(Q. 1-7)

The Action: In the second sentence of the opening paragraph, we’re told that there are six
chairs around a “circular table,” a big clue that this is a circle sequencing game. Our hunch
is confirmed when the intro goes on to identify our task: placing six trade reps around that
circular table. Even though circle sequencing games have been absent from recent LSATs,
one could show up on your test. Even if you don’t see a game exactly like this one, the
techniques and analytical skills you need to solve this type of game are similar, if not
identical, to those required for any game you’ll face. The Key Issues are:

1) Which representatives can, must, or cannot occupy adjacent chairs?


2) Which representatives can, must, or cannot sit between which other representatives?

In the majority of circle sequencing games, another key issue is who is across from whom.
How do we know that this game is not concerned with who’s across from whom? A quick
scan of the questions tells us. None of the question stems or answer choices include the
phrase “across from” or “opposite from.” Therefore, we know that we needn’t concern
ourselves with this element here.

Another important issue to look for in a circle sequencing game is the left/right issue. We
need to recognize whether the game is concerned with who sits to the left and right of
whom around the circle. How can we determine if this is an issue? A quick scan of the
game will tell us. Nowhere in the intro, rules, or questions do the words “left” or “right”
occur. As indicated clearly in the rules and questions, the game is concerned only with
who is next to and between whom.

© K A PL A N 19
LSAT PREP _________________________________________________________________ LSAT Test I Explained: Section II

The Initial Setup: There are two ways to lay out a circle sequencing game. We can either
draw a circle (and place the entities in chairs around it) or draw intersecting lines like the
spokes of a wheel (and place the entities at the ends of those spokes). The number of spokes
will depend upon the number of entities we’re placing around the table. The “spoke”
method is often preferable because it clearly shows not only which chairs are next to each
other, but also which chairs are across from each other. As we mentioned above, most
circle sequencing games are concerned with who’s across from whom, but scanning the
questions, it’s evident that this game isn’t. From the scan of the questions, it’s also evident
that even though the game numbers the chairs 1 through 6, none of the questions but the
first mentions a chair number. So we won’t bother numbering the chairs in our sketch:

K L MNOp

The Rules:

1) Pairs of entities are quite useful. Two entities that must always be together (or adjacent
in this game) give us a solid point from which to begin work on each question. Rule 1
provides us with P and N as such a pair. Under Key Issues, we already recognized that this
game isn’t concerned with who’s to the left or right of whom, so we don’t need to take that
into account when symbolizing the rules. “ALWAYS PN” will serve as a reminder that
these two must always sit together. (If this game did involve the left/right issue, we’d have
to write “ALWAYS PN OR NP.”)

2) This rule may have looked complex and confusing at first, but remember to ask yourself
what this rule means, not just what it says. After interrogating the rule in this way, it
hopefully appeared neither complex nor confusing. It simply says that L must always sit
next to M or N or both of them. Again, this game isn’t concerned with left/right, so “LM,
LN, or MLN” is a good way to keep this rule in mind.

3) Here we’re given a pair of representatives who can’t sit next to each other. Knowing who
can’t sit together isn’t as useful as knowing who must sit together (as in Rule 1), but this
type of rule does serve to limit the possibilities. K and M must never be next to each other
around the table. Write “NEVER KM” to serve as a reminder.

4) is an if-then rule, so be sure to take your time and carefully think through the rule and
its contrapositive. If O is sitting next to P, then O isn’t also next to M. We can get the

20 © K A PL A N
LSAT PREP _________________________________________________________________ LSAT Test I Explained: Section II

contrapositive simply by reversing these terms and negating them. If O is sitting next to M,
then O isn’t also next to P. “If OP, then no OM” and “If OM, then no OP” should keep the
rule and its contrapositive in our minds.

Key Deductions: The most important deduction here is recognizing that this game isn’t
concerned with who’s across from whom and who’s to the right or left of whom. These
realizations may not seem like enormous deductions, but they give us quite a leg up on
this game. Now, we don’t have to waste time worrying about those issues, and this also
leaves us greater flexibility to toss entities into the circle at the beginning of each question.
Furthermore, we can simply throw the P/N pair from Rule 1 into the circle as the starting
point when there seems to be no other logical place to begin (since we need not worry
about the order they take as long as they’re adjacent).

The Final Visualization: Here’s what we have as we go on to the questions:

K L MNOp

ALWAYS PN

NEVER KM

LM, LN, or MLN

If OP, then no OM

If OM, then no OP

© K A PL A N 21
LSAT PREP _________________________________________________________________ LSAT Test I Explained: Section II

The Big Picture:

• Time taken up front is especially important in all Logic Games, because a careless
master sketch can turn a relatively straightforward game like this into a nightmare. A
sketch, first and foremost, should be a tool to help you quickly and accurately
answer the questions and accumulate points. One instance of sloppy thinking
during the setup can foil both of these goals.

• With a sketch as simple as this one, you shouldn’t hesitate to redraw it as needed. It
would most likely take much more time to figure everything out in your head than it
would to quickly recopy the sketch.

• Not all games are ripe with deductions. However, just because a game doesn’t have
any earth-shattering deductions doesn’t mean that there’s no work to be done in the
setup. Make sure that you understand the game’s action and that you have translated
the rules into an accurate, accessible form.

• Pay attention to the “scope” of the game. Although this term is usually reserved for
Logical Reasoning, it applies here as well: The scope of a game includes the Key
Issues that will govern the game’s action as well as your work with the questions.
Anything else is outside the scope of the game, and you needn’t concern yourself
with such distractions. Here, we saw that the entire action is governed by “next to”
and “between” — “across from” and “left and right of” play no part. Recognizing
this, and narrowing your focus accordingly, makes the game that much easier to
handle.

• Sometimes a brief scan of the questions can be very useful, especially if you’re
struggling to understand the game’s Key Issues. Here, a quick scan of the questions
tells us which aspects of the typical circle sequencing game are important, and
which are not. Furthermore, if you scanned the questions during the setup stage,
you probably noticed that aside from Q. 1, the issue of naming the chairs 1 - 6 is
largely irrelevant, which may have saved you time in creating your sketch on the
page.

The Questions:

1. (B)
Oh good, here’s an acceptability question. Normally we’d just check each rule against the
choices and eliminate the choices that violate a rule. However, there’s something we need
to notice about this acceptability question. Thanks to the circular design of the table, the
first person of each list (in chair 1) is next to the last (in chair 6). For example, we need to
realize that, in choice (A), K sits next to L. With that in mind, we can treat this just like any
other acceptability question; just grab each rule and cross off any choice that violates that
rule. Rule 1 is violated by answer choice (E) which doesn’t have P next to N. Rule 2
requires that L be next to M, N, or both. Answer choice (A) has L next to O and K (L in seat
6 and K in seat 1) and so can be eliminated. Rule 3 forbids K to be next to M which answer
choice (D) tries (M in seat 6 wraps around and is next to K in seat 1). Finally, the

22 © K A PL A N
LSAT PREP _________________________________________________________________ LSAT Test I Explained: Section II

conditional Rule 4 is violated by answer choice (C) with has O next to both P and M.
Choice (B) remains and is the credited response.

• Acceptability questions are useful tools. First of all, they can give you an easy point.
Furthermore, they give you an opportunity to see first-hand how the rules and
entities interrelate. Finally, you are given one complete grouping that works (the
answer).

• This question is a great example of the importance of paying attention to non-


indented rules—those bits of information often hidden in the opening paragraph.
Here we were told that “successively numbered chairs” are adjacent, and chairs 1 and
6 are also adjacent. These rules are every bit as important as those of the indented
variety. Look for them.

2. (A)
The stem says that L sits next to P. Get this new information down on the page. The game
isn’t concerned with who’s to the left and right, so just make a new sketch and put L and P
next to each other. Where to now? Look for rules that have L or P in them. Rule 1 says that
P must sit next to N. L is on one side of P, so N must be on the other. Rule 2 says that L
must sit next to M or N or both. Here N is on one side of P with L on the other side, so M
must be next to L. Who’s left? K and O are left to occupy the remaining adjacent chairs in
between M and N. Rule 3 says that K can’t sit next to M, so K must sit next to N while O sits
next to M. The complete ordering is now set, so it’s a simple matter of skimming the
choices for the pair that must sit together. K and O must sit next to each other and (A) is the
answer.

• Get any new information down in your test booklet and see where it leads. Look for
rules that have the same entities as the new information in the stem. That is how
you’ll find the answer.

• In a typical “must be true” question, the wrong answer choices will be things that
either can, but need not, be true, or things that must be false. Here, that wasn’t the
case. All of the wrong choices had to be false. You knew this because we were able to
deduce the complete ordering around the table. This is helpful because knowing
what the wrong choices will look like allows you to eliminate them more quickly.

© K A PL A N 23
LSAT PREP _________________________________________________________________ LSAT Test I Explained: Section II

3. (B)
Once again, get the new information down and check the rules. We’re not concerned with
the “left/right” issue, so just draw a new sketch and put K between L and P. Now look for
rules with K, L, or P. Rule 1 says that P and N must always sit next to each other. K is next
to P on one side, so N must be on the other side of P. Our new sketch clearly shows that L
and N aren’t next to each other, which means it’s time to enact Rule 2: L must have M
beside her. Only one entity and one slot remain, so O will fill in the chair between M and
N. The new and complete sketch looks like this:

K
L p

M N
O
The question asks for the reps that M sits between, and that’s L and O, choice (B).

• Let your pencil work for you. In this question, we’re able to deduce everyone’s
placement from the information in the stem. All you had to do was dash out a quick
redrawing of the sketch, add the new information, and you were off to the races.
However, if you had tried to figure it all out in your head instead of making a new
sketch, it would have taken much longer, and you’d be much more likely to make a
mistake.

24 © K A PL A N
LSAT PREP _________________________________________________________________ LSAT Test I Explained: Section II

4. (E)
Once again, all you have to do is create a new sketch and add the new information: N is
next to M. By now you should be used to putting P and N together. M is on one side of N,
so P must be on the other. You should also be used to placing L next to M and/or N.
There’s no open chair next to N, so L must go next to M. O and K are left to fill in the two
chairs between P and L, and this time nothing forbids either of these people from taking
either of the remaining chairs. The sketch therefore looks like this:

N
p M

K/O L
K/O
The question asks which reps K can sit between. Depending on where K and O sit, K can
either sit between P and O or O and L. The testmakers chose the former in choice (E).

• Once you’ve created a complete and accurate master sketch, wean yourself away from
the actual rules as printed on the page. By this point in the game, you’re probably
used to always putting P and N together, always putting L next to M and/or N, and
always keeping K and M apart. All of this should be second nature, so why waste
time checking the actual printed rule each time you use it? You symbolized each rule
in your master sketch, so you should never have to read the actual rules again. The
key to this is making sure that your master sketch is accurate. You can’t depend
solely on your master sketch (and various redrawings of it) unless it’s totally
accurate. One mistake in your setup can cost you valuable points.

5. (E)
We’re helped immensely in this question by referring to our previous work. M is on one
side of L, and the question asks for a complete list of all of the reps who could be on the
other side. In the previous question, Q. 4, L was next to M on one side, and could be next to
either K or O on the other. This immediately allows us to eliminate (A), (B), and (C), none
of which includes O. (Incidentally, (C) is doubly wrong by not including K.) (D) and (E)
are left. How are these two choices different? (E) includes N while (D) does not. If we can
prove that N can be on the other side of L, then (E) is the answer; if not, we go with (D). Is it
possible for N to sit on the other side of L? Sure, here is the complete ordering:

© K A PL A N 25
LSAT PREP _________________________________________________________________ LSAT Test I Explained: Section II

O
K M

p L
N
Neri must be included, so (E) gets the point for Q. 5.

• When possible, use your work from previous questions to help eliminate choices in
later questions. Here, without doing any new work, you could eliminate three of the
four wrong answer choices. This is how you’ll have time to get to each question in
the section. A little time saved here and a little time saved there makes quite a
difference by the end of the section. And . . .

• Once you’ve narrowed down the answer choices, you can save more time by using
good answer choice strategies. When the choices contain lists, ask yourself “How do
these remaining choices differ?” If one choice contains an entity not contained in the
other remaining choices (like Neri in choice (E) above), you can save time by
checking for that entity.

6. (C)
Once again, don’t hesitate to quickly redraw the master sketch including the new
information from the stem (L next to N). L is on one side of N, so Rule 1 forces P to sit on
the other side of N. Where to now? We’re left with K, M, and O to fill in the remaining three
adjacent slots. We know from Rule 3 that K and M can’t sit next to each other, so they will
have to be separated by O. Either K or M will sit next to L while the other sits next to P—
and don’t forget O in between K and M, no matter which way K and M sit. With the
exception of K and M, this ordering is completely set. Now move on to the choices,
keeping in mind that we’re looking for the choice that must be false.

(A) must be true. K must sit between either O and L or O and P.

(B) could be true. M could sit between L and O with K sitting between P and O.

(C) No way. O can’t sit next to P. She must sit between K and M, as we deduced. (C) must
be false and is the answer.

(D) must be true. We deduced from the information given in the stem that N must sit
between L and P.

26 © K A PL A N
LSAT PREP _________________________________________________________________ LSAT Test I Explained: Section II

(E) must be true. As we saw in our original analysis and again in the context of correct
choice (C), O must sit between K and M.

• By Q. 6 in any game, the rules and deductions you’ve made should be second nature.
You should be used to placing P next to N and L next to M and/or N since you’ve
done this with every question so far. By this point, strive to be working with
confidence, sure of yourself and your deductions.

7. (E)
This stem tells us that K sits next to O. And we can’t take it any further. It looks like we’ll
just have to check each choice. But don’t despair! Use your previous work. Look for
instances where K and O sat next to each other and see which reps L was sitting between in
those situations. This question asks who L can’t sit between, so we can eliminate any choice
that is disproved by those previous instances. In the previous question, K was next to O,
and in that setup, L could sit between either K and N or M and N. Eliminate choices (B)
and (C). Where else was K next to O? In Q. 4, K and O sat together, and in that scenario we
saw it was possible for L to sit between K and M—eliminate choice (A). Finally, in question
2, K and O sat together, and L sat between M and P. Eliminate choice (D). Choice (E)
remains and is the answer.

• This question gave you a good clue that using your previous work was the way to go.
Even with the new information from the stem, you couldn’t make any deductions to
get even close to the answer. You had to just check each choice. When you see such a
question, chances are good that you can use your previous work to eliminate at least
some of the wrong choices—here, however, we were able to eliminate them all!

© K A PL A N 27
LSAT PREP _________________________________________________________________ LSAT Test I Explained: Section II

GAME 2 — Computers and Printers


(Q. 8-13)

The Action: After previewing the introductory paragraph and rules, we see that this game
asks us to match up four computers and four printers with the year in which each was
purchased—a matching game. The eight machines are in four offices numbered 1 through
4, one computer and one printer per office. The Key Issues will deal with:

1) In what year was each computer and printed bought?


2) What computer or printer can, must, or cannot have been bought earlier or later than
what other computer or printer?
3) What computer or printer can, must, or cannot have been bought the same year as what
other computer or printer?

The Initial Setup: One of the most important choices we have to make with this game is
deciding how to organize its information. As with most matching games, you can either
use a grid or lists to keep track of the info. Looking at this game, a grid might be slightly
more useful than the lists. Why? Well, look at the information. When deciding how to
organize a game, look for what is variable and what is definitely set. Here, the four offices
are definitely set, and one computer and one printer in each are definitely set. The only
thing that is variable is the date when each machine was purchased. So we have two fixed
points and one variable. That is a hint that a grid might be the best way to go. One fixed
point will go along the top of the grid and one will go along the side. We’ll fill the grid in
with the variable aspect, in this case, the date:

7 8 9

C P
1
2
3
4
Note that we shortened the dates from 1987, 1988, 1989 to just 7, 8, 9. This is a good time-
saving trick. The dates are all identical except for the last digit, so we’ll only concern
ourselves with that digit.

28 © K A PL A N
LSAT PREP _________________________________________________________________ LSAT Test I Explained: Section II

The Rules:

1) In each office, the computer was bought either earlier than or the same year as the
printer in that office. In other words, if the computer in 1 was bought in 1988, then the
printer in 1 must have been bought in 1988 or 89. Build this directly into the sketch. In
between each office’s C and P, draw a “£.”

2) Here’s information that connects the machines in two different offices. The computer in
office 2 and the printer in office 1 were bought the same year. Build this directly into the
sketch by drawing arrows between the boxes representing these two machines.

3) More inter-office information—the computer in office 3 and the printer in office 4 were
bought in the same year. Again, build this directly into your sketch by drawing arrows
connecting these two squares of the grid.

4) This time we’re given two machines bought in different years: the computer in office 2
and the computer in office 3. Build this information directly into the sketch by placing a
“X” on the line between the corresponding squares in the grid.

5) gives the only concrete information of the bunch. The computer in office 1 and the
printer in office 3 were both bought in 88. Build this right into the sketch.

Key Deductions: This game has a ton of entities and a ton of rules. Both of these facts are
hints that there’s some major deducing to be done here. So let’s get started.

Begin with the concrete info in Rule 5. The computer in office 1 was bought in 88. Rule 1
says that each office’s computer was bought earlier than or the same year as that office’s
printer. So the printer in office 1 must have been bought in 88 or 89. Add this to the
sketch.

Rule 2 says that the printer in office 1 was bought the same year as the computer in office 2.
Since we just saw that the printer in office 1 was bought in 88 or 89, so was the computer
in office 2. Add this to the sketch.

From there Rule 1 comes back into play. Since the computer in office 2 was bought in 88 or
89, the printer in office 2 must also have been bought in 88 or 89. Add this to the sketch.

Back to Rule 5. Since the printer in office 3 was bought in 88, the computer in office 3
(bought earlier than or the same year as the printer—Rule 1) must have been bought in 87
or 88. Add this to the sketch.

Since the computer in office 3 was bought in 87 or 88, we can combine this with Rule 3 to
deduce that the printer in office 4 must also have been bought in 87 or 88. Add this as
well.

Finally, since the printer in office 4 was bought in 87 or 88, Rule 1 means that the computer
in office 4 must also have been bought in 87 or 88. Add this to the sketch.

© K A PL A N 29
LSAT PREP _________________________________________________________________ LSAT Test I Explained: Section II

The Final Visualization: Here’s that ton of information added to the sketch:

7 8 9

C P
1 8 £ 8/9
2 8/9 £ 8/9
X
3 7/8 £ 8
4 7/8
£ 7/8

The Big Picture:

• This game is the quintessential example of why it’s vital that you take an appropriate
amount of time up front to work through all of the possible deductions. Here you’re
able to narrow down the possibilities to one or two years for all of the machines. This
will GREATLY simplify your work when you hit the questions.

• When you’re figuring out how to best organize the information in a matching game,
look for what is variable and what is fixed. The variable aspect is what can change,
and the questions will (pardon the expression) invariably ask you about this
variable aspect. Here, the year is the only thing that can change. The four offices are
set. The one computer and one printer in each office are also unmovable. When
using a grid, the fixed aspects go along the sides of the grid, and you’ll fill in the
squares of the grid with the variable aspect.

• Some students were hesitant to put in this much time during the setup. They
thought that it would be too much trouble to redraw this master sketch over and
over. However, with so much already deduced, you’ll find that you don’t need to
redraw very often: Unlike in Game 1, eyeballing the answer from the master sketch is
possible in many cases because we’ve already narrowed down the possibilities
considerably. Indeed, some students found that they only had to redraw the master
sketch for Q. 13 (which contains a rule change).

30 © K A PL A N
LSAT PREP _________________________________________________________________ LSAT Test I Explained: Section II

The Questions:

8. (B)
The printer in office 3 was bought in 88 (Rule 5). The only way for the computer in 3 to be
bought earlier is if it was bought in 87. Rule 3 says that the computer in 3 and the printer in
4 were bought the same year, so the printer in office 4 must have also been bought in 87.
Eliminate (D) and (E). Each office’s computer was bought in the same year as or earlier
than the printer in that office (Rule 1), so since the printer in 4 was bought in 87, the only
year that the computer in 4 could have been bought is also 87. Eliminate (C). (A) and (B)
are all that’s left. We deduced in the setup that the computer in office 2 must have been
bought in 88 or 89. Cross off (A), which leaves (B), the answer.

• Take your time with early questions and use them to get a good grasp on the game’s
action. Usually, early questions are slightly easier; use these questions to make sure
you understand what’s going on.

9. (D)
Here’s a could be true question with no new information. Normally this type of question
might take a lot of time to answer. Here, however, with all the good work we did up front,
we should be able to eliminate the wrong answer choices quickly. Just compare each
choice against our master sketch.

(A) No, the printer in 1 must have been bought in 88 or 89.

(B) No, the computer in 2 must have been bought in 88 or 89.

(C) No, the computer in 3 must have been bought in 87 or 88.

(D) Yes, the printer in 4 could have been bought in 88 (or 87). (D) could be true and is the
answer.

(E) No, the printer in 4 must have been bought in 87 or 88.

• Use the early questions to solidify your hold on a game. Here’s how: Let’s say you
didn’t deduce all of the machines down to one or two possibilities up front. Well,
here’s your opportunity to catch up. Since the right answer is a statement that
COULD be true, each of the wrong choices must contain a statement that CANNOT
be true. By testing choice (A) and proving to yourself that the printer in office 1
could NOT have been bought in 87 (wrong choice (A)), you’re forced to deduce that
the printer in 1 must have been bought in 88 or 89. The same holds true for choices
(B), (C), and (E). If you miss deductions up front, all is not lost—you can still pick
them up along the way in the process of working through the early questions.

© K A PL A N 31
LSAT PREP _________________________________________________________________ LSAT Test I Explained: Section II

10. (A)
We want as few machines bought in 87 as possible. Check our master sketch. There are only
three machines that could have been bought in 87. The computer in 3, the computer in 4,
and the printer in 4 all were bought in 87 or 88. The stem asks for the fewest number of
machines bought in 87, so could all three of these have been bought in 88? Sure. Rule 3 says
that the computer in 3 was bought the same year as the printer in 4, so both of these could
have been bought in 88. Rule 1 says that the computer in each office must have been bought
the same year as or before the printer in that office, so the computer in 4 could have been
bought in 88 as well. All of the other machines couldn’t have been purchased in 87 (see Key
Deductions), so it turns out that we need not have any 87 machines. In other words, the
fewest possible number of machines that were bought in 87 is 0, choice (A).

• Don’t assume that you must use your pencil for every question. Sometimes you can
answer a question simply by referencing the master sketch. That’s certainly the case
here. Recognizing when you don’t need to redraw the master sketch can save you
valuable time on Test Day.

11. (B)
Remember that the printer in each office was bought either the same year as or later than
the computer in that office (Rule 1). If the computer in 4 was bought in 88, the printer in 4
must also have been bought in 88 (we ruled out 89 as a possibility for the printer in 4 under
Key Deductions). We’ve just made a new deduction, so stop and see if that is an answer
choice. It’s not (although we can eliminate (E) which has the printer in 4 bought in 89), so
keep looking.

Rule 3 says that the printer in 4 and the computer in 3 were bought the same year, so the
computer in 3 must have been bought in 88. Is that a choice? No, but we can cross off (D)
which has it bought in 87.

Rule 4 says that the computer in 2 can’t have been purchased the same year as the
computer in 3, so here the computer in 2 must have been bought in 89 (again check Key
Deductions to see how 87 was eliminated for the computer in 2). Is “computer in 2 bought
in 89” an answer choice? No, but we now know that (C) must be false, so cross it off.

Moving along, Rule 2 says that the computer in 2 and the printer in 1 were bought the same
year, so the printer in 1 was bought in 89, and finally that’s the answer, choice (B).

• When given new information, follow the chain of deductions. Keep looking for
rules and deductions that lead you to the next entity and the next deduction.
Somewhere along this chain you’ll encounter the answer. Whenever you make a
deduction, though, stop and quickly see if that’s a choice. You don’t want to do more
work than is needed. Here that technique wasn’t as helpful as it normally is, because
the answer turned out to be the last stop in our logical chain. However, in “must be
true” questions, the strategy of checking the choices after making new deductions
will save you time more often than not.

32 © K A PL A N
LSAT PREP _________________________________________________________________ LSAT Test I Explained: Section II

12. (D)
If the computer in 3 was bought in 88, Rule 3 dictates that the printer in 4 was bought in 88
as well. Rule 4 (computer in 3 and computer in 2 bought in different years) means that the
computer in 2 was bought in 89. From there, Rule 2 (computer in 2 and printer in 1 bought
in the same year) insists that the printer in 1 was bought in 89. Also, since the computer in 2
was bought in 89, Rule 1 means that the printer in 2 was also bought in 89. The stem asks for
what could be true. The only machine that still has more than one option is the computer
in 4 which could have been bought in 87 or 88. Look first at any choice that includes the
computer in 4. Sure enough, choice (D), the computer in 4 being bought in 87, is the only
possible choice.

13. (C)
And here’s the one question that absolutely must have a new sketch. Now we’re to consider
what happens when the opposite of Rule 4 is true. The computer in 2 and the computer in
3 now are bought in the same year. Take a moment and create a new master sketch
including this new information. Look at the original sketch and note what information still
holds. The deduction that the printer in 1 was bought in 88 or 89 still holds, as does Rule 2
which means that the computer in 2 was also bought in 88 or 89. The stem says that the
computers in 2 and 3 were bought in the same year, so the computer in 3 must also have
been bought in 88 or 89. But wait. The printer in 3 was bought in 88, so Rule 1 means that
the computer in 3 can only have been bought in 88. This also means that the computer in 2
was also bought in 88 (thanks to the new information in the stem). From there, Rule 2 means
that the printer in 1 was bought in 88 as well. Since the computer in 3 was bought in 88,
Rule 3 forces the printer in 4 to have been bought in 88. The only machines that possibly
weren’t bought in 88 are the printer in 2 (88 or 89) and the computer in 4 (87 or 88). The
printer in 2 is the only machine that could have been bought in 89, choice (C).

• Normally a question that changes or adds a rule takes a lot of time and would be an
excellent candidate for skipping the first time around. However, many students
found that they had such a good handle on this game that they were able to make the
change and could almost automatically see how that change affected the numbers.
The point is this: Be flexible. If you’ve got such a great handle on a game that you
can make a rule change almost all in your head, then do it.

© K A PL A N 33
LSAT PREP _________________________________________________________________ LSAT Test I Explained: Section II

GAME 3 — Law Firm Hirings


(Q. 14-18)

The Action: After conducting an overview of this game, we see that we’re asked to put
eight partners in order based on the year in which they joined the firm—a sequencing
game. Moreover, it’s a “free-floating” sequencing game. We call it this because all of the
entities are described relative to each other. In “free-floating” games, the important aspect is
where each entity is relative to the others—Hodges is before Nader, Owens is after Nader,
etc. Notice here that a range—1961 to 1968—is given for the entities. But this is still a “free-
floating” game since the entities are only listed before or after each other and not in specific
years. The Key Issues will be:

1) When did each partner join the firm?


2) What partners can, must, or cannot have joined the firm before and after what other
partners?

The Initial Setup: With most “free-floating” sequence games, the most natural way to
work is vertically. Put the years along the side and list the partners. Notice that, as in Game
2, we’ve shortened the years to just 1 through 8. (You could also have worked horizontally
by listing the years left to right like a timeline. Whichever way you choose, just make sure
that your sketch is neat and not misleading.)

GHIJKMNO
1
2
3
4
5
6
7
8

The Rules:

A good way to start is to locate a partner or partners who might come at the top of the
sketch; that is, anyone who doesn’t seem to have joined the firm after other partners. If we
scan the rules, we see that H in Rule 1 and K in Rule 2 both fit that description. Let’s
therefore start by putting an “H” and a “K” at the top of the sketch.

34 © K A PL A N
LSAT PREP _________________________________________________________________ LSAT Test I Explained: Section II

1) H joined before N. From the H at the top of the sketch, add a line down to an N.

2) K joined the firm before J. Next to the H at the top of the sketch, we already have a K.
From that K draw a line down to a J.

3) N and J both joined the firm before G. There’s already an N and a J in the sketch, so draw
lines down from each of them to a G.

4) gives more information about N. N joined the firm before O. From the N in the sketch,
draw another line down to an O.

5) Here’s more information about J. J joined the firm before M. From the J in the sketch,
draw another line down to an M.

6) G joined the firm before I. From the G in the sketch, draw a line down to an I.

Key Deductions: The first thing to do is check and make sure that all of the entities are
included in the master sketch. If one isn’t, that’s a “floater” that can be plugged in
anywhere. Fortunately, all of the entities are included in the sketch, so we can depend
solely on our master sketch to answer every question we’ll face.

Before moving on, take a moment and review the sketch. Some things should be obvious.
For instance, K joined the firm before G, I joined the firm after N, etc. Also take note of the
possibilities regarding the specific years; what entities could have joined the firm in 1961?
What entities could have joined in 68? H and K are the only entities that aren’t below an
entity, so either H or K must have joined in 61. I, O, and M are the only partners with no
entities below them, so they’re the only ones who could have joined in 68. While these
aren’t earth-shattering deductions, it’s a good idea to take a moment and review what the
completed sketch can tell you. This sets up the game’s parameters in your mind.

It’s also wise to take a moment and recognize what the sketch can’t tell us. Be careful not to
assume any relationship between partners that aren’t connected by lines. For example, I, in
our sketch, is slightly lower than M and O. But we can’t assume that M and O joined the
firm before I. We don’t know that. Be very careful when dealing with the two “arms” of the
sketch. The “K—J—M” arm is not connected to the “H—N—O” arm. Therefore, we can’t
deduce any relationship between the two. M, for instance, is quite low on the sketch, but
since it’s not connected to the “H—N—O” arm, M could have joined the firm even before
H. Likewise, O could have joined before K.

© K A PL A N 35
LSAT PREP _________________________________________________________________ LSAT Test I Explained: Section II

The Final Visualization: Be sure you’ve kept all of your notations neat and easily
accessible:

GHIJKMNO
1
H K
2
3
N J
4
5 G
6 O M
7 I
8

The Big Picture:

• It’s extremely important to review the section before you dive in. Most students
found this game, the third in the section, to be the most straightforward. You want to
search out the most familiar game and start your section there. You’ll give yourself a
jump-start with the points and confidence this game should provide.

• Don’t infer relationships that don’t exist. Don’t think that M must have joined after
H. On the contrary, M could have joined before H, N, and O.

The Questions:

14. (C)
In a “free-floating” sequence game like this (when all the entities are in the master sketch),
just check each choice against the sketch, using it to figure out the choice that is
impossible.

(A) H (or K) could have joined the firm in 61. Answer choice (A) can be true and isn’t the
answer.

(B) If K joined in 61 and J in 62, then H could have joined in 63. (B) can be true and isn’t the
answer.

36 © K A PL A N
LSAT PREP _________________________________________________________________ LSAT Test I Explained: Section II

(C) Four entities, H, K, J, and N are all above G in the sketch, so at least four partners
joined the firm before G. The earliest that G could have joined the firm is 65. Answer
choice (C) can’t be true and is the answer. Quickly, here is why choice (D) and choice (E)
can be true.

(D) M could have joined the firm anytime from 63 (K and J joined earlier) to 68.

(E) O could have joined the firm anytime from 63 (H and N joined earlier) to 68.

• Usually a “cannot be true” question with no new information takes quite a long time
to answer, and you may have considered skipping this one the first time around.
Here, however, you were able to solely depend on the master sketch to quickly find
the answer.

15. (E)
Again, we can solely depend on the sketch. If J joined in 62, exactly one partner joined
earlier (the one that joined in 61). Looking at the sketch, K must have joined the firm earlier
than J, so K joined in 61. That’s all we know for sure. Either H or M could have joined in 63,
either H or M or N could have joined in 64, and so on. Check each choice looking for the
one that can’t be true.

(A) and (B) Either H or M could have joined in 63 after K (61) and J (62).

(C) Could H have joined in 64? Sure, here is the ordering: K, J, M, H, N, G, I, O.

(D) Could N have joined in 64? Sure, here is the ordering: K, J, H, N, M, G, I, O.

(E) K joined in 61 and J joined in 62. At least H and N joined before O, so the earliest O
could have joined is in 65. (E) can’t be true and is the answer.

16. (D)
We’re asked for the latest year that J could have joined the firm. There are two ways to go
about this: We can put as many partners as possible before J, or we can see which partners
MUST be after J. Since the latter deals with what is DEFINITELY true while the former
deals with what is POSSIBLE only, the latter is most likely going to be simpler and, more
importantly, quicker. Looking at the sketch, only G, I, and M are definitely placed below J.
Therefore if G, I, and M joined the firm in 66, 67, and 68, the latest that J could have joined
the firm is 65, choice (D).

• There is often more than one way to tackle a question. Strive to choose the tack that
is the quickest and seems most natural to you. Getting the point is of the primary
importance, but getting the point as quickly and efficiently as possible is vital if
you’re going to get to every question on the section.

© K A PL A N 37
LSAT PREP _________________________________________________________________ LSAT Test I Explained: Section II

17. (B)
From the master sketch, we’re able to deduce that either I, M, or O must have joined the
firm last, in 68. This question stem assigns O to 65 and M to 67. Therefore, I must have
joined the firm in 68. Now check the master sketch. H and N joined before O in the 61-64
slots. K and J joined before M and also before G. Since H and N must occupy two of the
four 61-64 slots, K and J will take the other two slots which leaves G (which must go AFTER
K and J) to take the remaining 66 slot. G and I are the only other entities whose slots can be
determined, choice (B).

• Never relax your critical thinking skills. The question stem asked for the OTHER
entities not counting M and O whose positions can be determined. If you glossed
over this fact and counted M and O in your final tally, your work would have been
wasted and you would have missed this point.

18. (D)
Here we’re to assume that O joined the firm before M, and we’re looking for the earliest
that M could have joined. Since H and N joined the firm before O, all three of these must
have joined the firm before M. K and J joined the firm before M (as always), so that makes a
total of five partners that must have joined the firm before M. These five will at least take
up the 61-65 slots, so the earliest that M could have joined the firm is 66, choice (D).

• This is the ideal game with which to start your section. It’s probably the easiest game
of the bunch and its action is familiar. By beginning your day here, you ease
yourself into the section and gain confidence (and the points!). In addition, most
students can do this game in well under 8 minutes, so they’ve already got extra time
to use on a later, tougher game.

38 © K A PL A N
LSAT PREP _________________________________________________________________ LSAT Test I Explained: Section II

GAME 4 — Railway Tickets


(Q. 19-24)

The Action: After reading the opening paragraph and rules, we see that this game requires
that we match up the January and February tickets for three railway lines with the color of
each—a matching game. The action doesn’t become fully apparent until Rule 1 which
introduces the concept of the “color” of each set of tickets. Some students were thrown by
the notion of “sets” of tickets in the opening paragraph. They were worried that this
implied a whole lot of tickets. If we read carefully, though, we know that that’s not a
problem. There is one “set” of tickets per month, as simple as that. The Key Issues are:

1) What color is each of the six sets of tickets?


2) What sets of tickets can, must, or cannot be the same color as what other sets of tickets?

The Initial Setup: As with Game 2 and other matching games, a grid or lists both work
well. In Game 2, we talked about identifying the variable and fixed aspects when using a
grid. What are the variable and fixed points in this game? The two months, January and
February, are fixed. They don’t change. The three railway lines are also fixed, not going
anywhere. What can change from line to line and month to month is the color of the sets of
tickets. This is the variable aspect. So when creating our grid, the two fixed points (months
and lines) will go along the top and side, and we will fill in the grid squares with the
variable (color):

Jan Feb
1 G P R Y

© K A PL A N 39
LSAT PREP _________________________________________________________________ LSAT Test I Explained: Section II

The Rules:

1) gives the information that we’ve already included in the sketch above, namely that each
set of tickets will be green, purple, red, or yellow.

2) Let’s take our time and make sure that we fully understand this rule. The January set of
tickets of each line (1, 2, and 3) will be a different color from the February set of tickets of
that line. Let’s do a “what if” to make sure we have a handle on this rule. If the Jan 1
(January line 1) set of tickets is red, then the Feb 1 (February line 1) set of tickets cannot be
red. Draw a big “X” between each line’s January and February squares in our grid.

3) Again, we need to take our time when decoding this rule. Within each month, the set of
tickets of each line must be a different color than the set of tickets of the other two lines.
Let’s do another “what if.” If the Jan 1 (January line 1) set of tickets is red, then the Jan 2
(January line 2) set of tickets can’t be red. Between each month’s 1 and 2 squares draw a big
“X,” and between each month’s 2 and 3 squares draw a big “X.”

4) Of the three sets of January tickets, exactly one must be red. Build this directly into our
grid. Write “ex. 1 R” at the bottom of the January column.

5) Once more, take care when translating Rule 5. This rule is quite similar to Rule 4. In
Rule 4, exactly one of the sets of January tickets (either line 1, 2, or 3) was red. Here, exactly
one of the sets of line 3 tickets (either January or February) must be green. At the end of the
line 3 row, write “ex. 1 G.”

6) Here is a nice, concrete piece of information. The Jan 2 set of tickets is purple. Build this
right into the master sketch.

7) Finally, Rule 7 states that none of the sets of February tickets are purple. Write “NO P”
in each of the February squares in our grid.

Key Deductions: Rule 6 said that Jan 2 is purple. Rule 3 said that within each month, the
sets of tickets for the three lines must be of different colors. So we can deduce that Jan 1
and Jan 3 can’t be purple. Rule 7 already said that NO sets of February tickets are purple,
so we definitely know that Jan 2 is the only set of tickets that is purple.

We can deduce something else from Rule 7 which says that none of the February tickets are
purple. Rule 3 says that each month’s set of tickets must be different colors. Since none of
the February tickets are purple, that means that all three of the other colors must be used
exactly once for the three sets of February tickets, one green, one red, and one yellow.

40 © K A PL A N
LSAT PREP _________________________________________________________________ LSAT Test I Explained: Section II

The Final Visualization: Here’s our master sketch:

Jan Feb
no P no P
1 X G P R Y
X noXP
2 P X
X X
no P no P ex. 1
3 X G
ex. 1
R
The Big Picture:

• Usually, from the introductory paragraph alone, you’re able to get a handle on a
game’s action and create an initial setup. If you’re having trouble doing that,
sometimes you need more information found in the rules. In these cases, don’t
hesitate to read the first few rules to see if they can help cement the action. Here the
action didn’t become apparent until Rule 1.

• Organize the information in a way that’s easiest for you. We used a grid here, but
many people prefer to use lists. As long as you organize the information in a way
that is neat, accurate, and readily accessible, the form you choose is up to you.

The Questions:

19. (E)
Rule 5 says that one of the sets of line 3 tickets (either January or February) must be green.
The stem says that Jan 3 is red, so Feb 3 must be green, choice (E).

• After a time-consuming setup, don’t be surprised if the first question is fairly


simple. The testmakers like to reward you for taking an appropriate amount of time
up front to reason out the rules and organize the data.

20. (A)
What must be true if one of the sets of line 2 tickets is green? Jan 2 is purple (Rule 6), so the
only 2 line left to be green is Feb 2. Is Feb 2 green an answer choice? Nope, keep going.
Rule 3 says that no two sets of tickets in the same month can be the same color. Since Feb 2
is green, Feb 3 can’t be green. Rule 5 (either Jan 3 or Feb 3 must be green) comes into effect
again and Jan 3 must be green. Is Jan 3 green a choice? No, so keep looking. Rule 4 says that

© K A PL A N 41
LSAT PREP _________________________________________________________________ LSAT Test I Explained: Section II

one of the January tickets is red, and since Jan 2 is purple and Jan 3 is green, Jan 1 must be
red. And that’s choice (A) and the answer.

21. (A)
There’s not much to do with this “could be true” question with no new information but try
out each choice.

(A) Could none of the January tickets be green? If none are, we’re left with purple, red,
and yellow as the colors of the three sets of January tickets. Jan 2 is purple (Rule 6). Jan 1
could be red, and as long as Feb 3 is green (to fulfill Rule 5), Jan 3 could be yellow. Answer
choice (A) could be true and is the answer. Quickly here is why the remaining choices can’t
be true:

(B) Since none of the sets of February tickets are purple (Rule 7), exactly one set of the
February tickets MUST be green.

(C) The Jan 2 tickets are purple (Rule 6) and one set of the January tickets is red (Rule 4), so
either Jan 1 or Jan 3 must be red.

(D) Jan 2 is purple, so if one set of January tickets was green and the other set of January
tickets was yellow, there wouldn’t be any set of January tickets that was red which would
violate Rule 4.

(E) Jan 2 is purple, and Rule 7 says that absolutely none of the sets of February tickets can
be purple.

• Negative choices like (A) and (B) are a little slippery. You have to see if it could be
true that something doesn’t happen. Compare this with positive choices like (C), (D),
and (E). It’s simpler to check whether or not something positive could be true. For
this reason, you may have been better served by skipping Q. 21 in favor of Qs. 22 and
23 which have all positive answer choices (especially Q. 23 with its positive, short
answer choices).

22. (B)
Another “could be true” question with no new information (but with positive choices, at
least—see Bullet Point to Q. 21 above). Again, not much to do with this one but try out
each choice.

(A) If Jan 1 and Feb 2 are both green, Rule 3 means that none of the remaining sets of tickets
can be green. But Rule 5 insists that either Jan 3 or Feb 3 be green. (A) can’t be true and isn’t
the answer.

(B) Can Jan 1 and Feb 2 both be yellow? Sure: Jan 1—Y, Jan 2—P, Jan 3—R; Feb 1—R, Feb
2—Y, Feb 3—G. (B) can be true and is the answer. Quickly, here is why the remaining
choices can’t be true:

42 © K A PL A N
LSAT PREP _________________________________________________________________ LSAT Test I Explained: Section II

(C) If Jan 1 is yellow, the only set of January tickets left to be red (as Rule 4 requires) is Jan
3. Rule 5 then requires that Feb 3 be green, not yellow.

(D) If Jan 1 is green, Rule 3 means that none of the other sets of January tickets are green, so
Rule 5 again requires that Feb 3 be green, not red.

(E) If Jan 3 is yellow, the only set of January tickets left to be red (as Rule 4 requires) is Jan
1. Since Jan 1 is red, Rule 2 doesn’t permit Feb 1 to also be red.

23. (E)
If Feb 3 is yellow, Rule 5 forces Jan 3 to be green. Since Jan 3 is green, Jan 1 is left to be red
(as Rule 4 requires). Since none of the sets of February tickets are purple, Feb 1 and Feb 2
must be green and red. Since Jan 1 is red, Rule 2 means that Feb 1 can’t be red and must be
green. Feb 2 is left to be red. The entire setup is fixed: Jan 1—R, Jan 2—P, Jan 3—G; Feb 1—
G, Feb 2—R, Feb 3—Y. The only choice that doesn’t agree with this setup is (E)—only one of
the sets of tickets is yellow.

• Just by reading the question stem, you should have recognized that this question
would probably lead to all colors being set for every line’s tickets. Why? The phrase
“must be true EXCEPT” was your big clue. In order to come up with four wrong
answer choices full of must be trues, you can bet that you’ll need to keep looking for
deductions until you’ve got a near complete setup, if not a totally complete setup.

24. (C)
For this question, none of the sets of tickets are purple. This doesn’t affect the sets of
February tickets since none were purple anyway (Rule 7). But since none of the sets of
January tickets are purple (sorry Jan 2, you’re not purple anymore), one will be green, one
red, and one yellow. These three colors are the only options for both month’s tickets. Now
check the choices looking for the one that can be true.

(A) and (B) We can kill both of these choices immediately. Since none of the sets of tickets
are purple, all of the other colors must be used exactly once per month. Exactly one set of
January tickets and exactly one set of February tickets must be green.

(C) Could neither of the sets of line 2 tickets be green? Sure, here is the complete setup: Jan
1—Y, Jan 2—R, Jan 3—G; Feb 1—G, Feb 2—Y, Feb 3—R. (C) could be true and is the answer.

(D) If none of the sets of line 1 or line 2 tickets are yellow, then both Jan 3 and Feb 3 must be
yellow which Rule 2 forbids.

(E) If none of the sets of line 2 or line 3 tickets are red, then both Jan 1 and Feb 1 must be
red which Rule 2 forbids.

• In Q. 21, you saw two negative choices. In this question, all five choices are phrased
in negative terms. If you were running short on time at the end of a section, Q. 23
with its positive choices would be a much better place to look for a point than Q. 24
(or Q. 21) with its negative choices.

© K A PL A N 43
SECTION III:
LOGICAL REASONING

44 © K A PL A N
LSAT PREP ________________________________________________________________ LSAT Test I Explained: Section III

1. (A)
It’s tough to pre-phrase an answer to this one; you simply need to attack the choices in
search of an explanation as to why packing a wound with sugar would so successfully kill
those hardy and ubiquitous bacteria. And you needn’t search long: If sugar makes a
wound dry and if bacteria like moisture, as (A) states, then it’s not surprising that a sugar
environment tends to spell R.I.P. for bacteria.

(B) So sugar is available for use. But how would one use it? Why would it be successful?

(C)’s phenomenon creates a paradox, not resolves one. We’re trying to figure out why sugar
in a wound kills bacteria, not how it might provide an environment in which they thrive.

(D) deals with foods that contain sugar, which is far removed from the treatment described.
And in any case, if sugar were to weaken an antibiotic (assuming that such a drug were
prescribed for a wound), that would be harmful rather than helpful. Both (C) and (D) give
us more or less the opposite of what we want.

(E) is unnecessary background information. What does the long-time use of sugar on
wounds have to do with why it has the effect it does?

• When trying to strengthen or justify an argument or process, beware of answer


choices (such as (C) and (D), here) that tend to do the opposite of what you seek. We
call such choices “au contraire” and they are common on the LSAT.

2. (B)
This parallel reasoning stimulus can be easily translated in algebraic terms: If color-blind
(If X), then can’t differentiate green and brown (then Y). Gerald can’t differentiate those
colors (Since Y), so he must be color-blind (therefore X). This is a common logic error; it’s
called the fallacy of affirming the consequent—you’re not allowed to simply transpose the if
and then clauses of a conditional statement. (B) is set up in the same way: X is “suffering
from sinusitis,” Y is “losing one’s sense of smell,” and Mary plays the role of Gerald.

(A) and (D) represent a straightforward “If X, then Y; William (or Arthur, depending) is X,
therefore he’s Y.”

(C) is proper logic. Those who have had jaundice X can’t give blood Y, and so—here comes
the contrapositive—since Jean gave blood not Y, she can’t have had jaundice not X. That’s
absolutely valid and not parallel to the original.

(E) “Since X, therefore Y; Frieda’s X, therefore Z.” Where’d that Z term come from? (E)’s not
parallel either.

• When a Parallel Logic stimulus is written in strict formal terms as this one is, you
can easily and carefully translate it into X/Y or A/B terms, translate the five choices,
and find the one and only choice that matches.

© K A PL A N 45
LSAT PREP ________________________________________________________________ LSAT Test I Explained: Section III

3. (E)
Wegener’s continental drift theory is widely accepted, we’re told, despite the fact that we
don’t know why the continents move. We do however know that they move—the motion
has been confirmed—and that is evidently good enough for the scientific community. Each
element of (E) matches up to the passage: The “theory” is Wegener’s; the events postulated
are continental movements; and the explanation for same is what the author has told us is
missing.

(A) illustrates why science might accept a single theory that defines all of nature, but such
a theory is not Wegener’s. (A) fails to explain why this one particular theory has caught on
despite the absence of some key evidence.

(B) The “mathematical nature” stuff has no counterpart in the passage, nor does the issue
of science’s overall accuracy.

(C) Well, the one measuring improvement we hear about—the one that allows science to
confirm the movement of continents—has made it easier, not harder, for Wegener’s theory
to take hold.

(D) evokes an irrelevant distinction between single events vs. mass behavior, one that has
nothing to do with the topic and scope of the stimulus discussion.

• This is the LSAT’s very first “principle” question—the now common type in which
we have to match up a concrete situation to an abstract principle or statute, or
occasionally, vice versa. With such questions, you must pay careful attention to topic
and scope—you need to match up each key element of the situation to the statements
in the choices.

4. (D)
According to the author, the basis of the deterrence theory is that fear of retaliation keeps
(deters) a potential aggressor from attacking. She further argues that maintaining deterrence
requires maximum fear—that is, a potential aggressor must be so convinced of the
invincibility of its potential opponent that it will lay low. From that, (D) must follow: A
powerful nation seeking deterrence will want to let would-be aggressors know just how
strong it really is; that knowledge is what makes deterrence work.

It’s unlikely that you pre-phrased (D); what’s more likely is that after studying the
stimulus, you proceeded to examine the five choices and (with luck) chose the right one.
Here’s why the others should have been bypassed:

(A)’s necessary condition—the “certain knowledge” of total destruction, signaled by “only


if”—is not something to which the author is committed. Deterrence simply requires enough
awareness of an opponent’s strength to make the potential aggressor think twice. (A) is too
extreme to be a proper inference.

46 © K A PL A N
LSAT PREP ________________________________________________________________ LSAT Test I Explained: Section III

(B) offers a case in which military deterrence would not be in effect, because the aggressor
nation believes itself to be mightier than its potential target. This doesn't mean such a
nation will definitely attack the other nation for this reason alone, but we certainly can't
infer, as (B) does, that this country won't attack.

(C) Oh yeah? An equally likely reason that one nation fails to attack another is that it (the
first nation) simply isn’t warlike and has no designs on the other. (C) as written—implying
that all peace is based only on fear of retaliation—is deeply cynical at best, loony at worst.

(E) makes a classic LSAT scope shift. The stimulus says that deterrence relies on aggressors
believing that a nation has superior retaliatory power. (E) changes this to read that the
defending nation must actually possess that power, a subtle but very important difference.

• Pre-phrasing, as a technique, is less useful in inference questions than in other


types.

• When attacking inference questions, avoid answer choices, such as (A) here, that are
too extreme.

5. (C)
The conclusion that we’re directed to make Clay “reconsider” is signaled by the phrase
“This indicates that...,” and the gist of it is that it’s a rebuttal: For Clay, life as we know it
exists nowhere but on Earth, despite the astronomer’s prediction that lots of life is to be
found on planets Out There.

Notice that Clay spends most of his letter describing the astronomer’s thought process. He
provides only one piece of counter evidence, and it’s this: No single planet outside of our
own solar system has been found. On that absence of evidence hangs Clay’s stubborn,
categorical insistence that We Are Alone. Well, he’d have to back off if, as (C) states, the
reason that no other planets have been found is not that they don’t exist, but that we don’t
yet have the means to find them. If (C) is true, then until such time as adequate means of
detection is developed, the astronomer’s prediction is unscathed and will have to stand.

(A) sounds like a capitulation on Clay’s part, but it’s really outside the scope. The point at
issue between Clay and the astronomer is whether life as we know it exists elsewhere than
Earth. Clay could concede the existence of (A)’s weird life forms and still argue that the
astronomer is wrong.

(B) Planets and a star like the Sun are, to the astronomer, necessary for life as we know it to
exist. That some stars may be planet-less has no effect on either the astronomer’s logic or
Clay’s.

(D) is something that Clay already accepts as true—his whole rebuttal hinges on his belief
that it’s the astronomer’s faulty assumption (about other planets) that renders the
prediction worthless. Since (D) is in synch with Clay’s reasoning, it cannot serve to make
him reconsider that reasoning.

© K A PL A N 47
LSAT PREP ________________________________________________________________ LSAT Test I Explained: Section III

(E) is irrelevant. So communication between us and E.T. would be tough, eh. Would it be
impossible? What has this to do with whether life as we know it does or does not exist?

• Not everything in a stimulus argument is germane to the logic. Here, as noted above,
Clay Moltz’s argument is confined to the last two sentences; the rest of his letter is
background information and filler. Always endeavor to pull the evidence and
conclusion out, and paraphrase them, for maximum comprehension.

6. (A)
The conclusion is that since a necessary condition of Max’s guilt (the unwillingness to ask
the cops to investigate) is not met—in fact Max invites investigation—then he cannot be
guilty. In the same way, “my” inability to see Lucille in (A) is necessary in order for her to
be in the next room; since “I” haven’t met this necessary condition (i.e. I can see her), then
she must not be in that room. Yes, it’s simply classic if-then/contrapositive logic.

(B), (C) Each flips (transposes) its if-then statement: If rich, then no Alaskan vacation; since
no Alaskan vacation, therefore rich (B). If over 40, then no interest in learning; since no
interest in learning, therefore over 40 (C). The stimulus contraposes, rather than transposes,
its conditions, so neither (B) nor (C) is parallel.

(D) negates its if-then statement: If good cook, then no cinnamon added; since not a good
cook, cinnamon added. Again, to contrapose—which is what the stimulus and (A) do—
means to negate and transpose simultaneously.

(E) is straightforward enough: If one is sociable, then one won’t avoid her friends; Sally’s
sociable, therefore she’s not one to avoid her friends. This would work if it concluded
“Since Sally avoids her friends, she must not be sociable.”

• Having trouble with the concept of the contrapositive? Just study this question and
correct choice (A) from time to time. Each of them has it exactly right.

• Use these wrong choices as illustrations of the two most common fallacies: (B) and
(C) affirm the consequent, whereas (D) denies the antecedent. Both are commonly-
tested.

7. (A)
If time is tight—or you find that you cannot follow the argument as presented—this one is
practically screaming, “Skip me for now; come back to me later on.” Handling it requires a
firm grasp on the comparisons offered, as well as on the direct relationship that is the
author’s conclusion. Let’s start with that. According to the author, one will find a higher
percentage of older male ducks when there is a great disparity in sex ratios overall (i.e. all
males : all females). How come? Because among older ducks there are a lot more males,
whereas among younger ducks the male : female ratio is much closer to 1 : 1. In other
words, the more the male ducks strongly outnumber the females, the likelier it is that the
older males will outnumber the younger ones.

48 © K A PL A N
LSAT PREP ________________________________________________________________ LSAT Test I Explained: Section III

Compare the male : female ratio at the two lakes in question. The western lake’s is close to 1
: 1 (55 : 45), while the eastern lake’s is almost 2 : 1 (65 : 35). Which lake, according to the
conclusion, should have a greater percentage of older males? The eastern—that’s the lake
with the greater male : female disparity. So the western lake should have a lower percentage
of older males . . . and that’s just what (A) points out.

(B) The data in the argument compare the ages of adult and nonadult males. We cannot
draw (B)’s conclusion about adult vs. nonadult ducks—males and females combined.

(C) No conclusion about a total population can be drawn from a male : female ratio.
Contrary to (C), the western lake could have many more male ducks than the eastern, so
long as the western’s population reduces to 55 : 45 and the eastern’s to 65 : 35.

(D) introduces an issue—hatching time—that is totally removed from the argument. Hence,
(D) is completely unacceptable as an inference.

(E)’s conclusion about the age ratio of adult females : nonadult females is unjustified,
because no data comparing those groups are provided. The conclusion only concerns
males.

• Remember the high “truth threshold” that an LSAT inference must meet. It’s got to
be something that is certain, based on the text. Don’t settle for anything less!

• If a question strikes you as complex, don’t just bite the bullet and stop to work it all
out, no matter what. Be ready and willing to skip it happily and immediately, eager to
move ahead to more gettable points. Let’s face it, if you’re going to leave questions
undone, better they be really tough questions than easier ones. And in any case,
even really tough ones will look easier toward the end of the 35 minutes, when
you’ve already amassed a whole lot of points.

8. (B)
There’s a huge scope shift here—did you see it? This author argues that Yellow Dye No. 5
should be permitted because its health benefits outweigh its health risks. But the only
benefit cited is that the yellow color makes people enjoy lemon soda more. That’s a matter
of personal preference only! Yet the risk cited—the potential allergic reaction—is a real
health hazard. By the author’s own logic, any decision on a ban must be based on the
comparison of health benefits and health risks, and so the author has used inappropriate
evidence to make her recommendation. That’s the essence of (B).

(A) is flat wrong. The author flatly states that such a risk does exist.

(C)’s assertion that most people may be harmed by additives in general is irrelevant to the
discussion of this particular additive and its benefit/risk ratio.

(D) The benefit/risk ratio that the author recommends evaluating doesn’t hinge on risks
being proven, so (D) cannot explain the argument’s weakness.

© K A PL A N 49
LSAT PREP ________________________________________________________________ LSAT Test I Explained: Section III

(E) Consumer awareness is way outside the scope of the argument which concerns the
standards for determining whether or not an additive should be banned.

• To attack an argument, one must stay within the scope of the argument as its author
has laid it out. Note how most of the incorrect choices depart markedly from the
scope here. Departing from the argument’s scope is a sure sign of a wrong answer.

9. (A)
The conclusion that we are to weaken is a prediction: that businesses at risk of suffering
environmental accidents will now install safeguards, on the seemingly sensible rationale
that it’s now more expensive to pay a fine than to prevent an accident in the first place. But
that assumes, of course, that those businesses are prudent rather than gamblers by nature. If
(A) is true—if businesses typically do not acknowledge the risk of accident—then why
would they start installing safeguards as the author predicts? They’d be far more likely to
do nothing and take their chances. (A) severs the connection between evidence and
conclusion pretty effectively.

(B) is yet another irrelevant distinction—we’ve seen so many in the section so far, and
indeed many Logical Reasoning wrong answers feature such. The argument makes no
mention of long- vs. short-term business issues.

(C), if true, leaves us wondering whether the argument is damaged or strengthened. If, as
(C) says, making good business sense is a necessary condition of business’s doing the right
thing, environmentally speaking, we’re left to ask the question, “Does planning ahead for
potential disaster make good business sense or doesn’t it?” If Yes, the argument is
strengthened; if No, it’s weakened.

(D) is implying that businesses can write off a fine as if it’s nothing, and so they wouldn’t
bother with risk prevention. However, (D) still leaves open the possibility that a fine, even
seen as an “ordinary business expense,” might be so much bigger than the cost of
prevention that the author’s prediction would remain in effect.

(E) Business self-promotion and public awareness—however relevant to the general issues
raised—are irrelevant to the conclusion drawn.

• Remember, we can only strengthen or weaken an argument on the arguer’s own


terms. Choices like (B) and (E), which have nothing to do with those terms, cannot be
correct.

10. (C)
This one has a weird question stem. “Necessary to rely on” is LSAT code for “a necessary
assumption.” What they’re getting at is that four of the choices are assumptions necessary
to the reasoning; it follows that the right answer must either be outside the scope, or
contradict the text.

50 © K A PL A N
LSAT PREP ________________________________________________________________ LSAT Test I Explained: Section III

The issue is this: How freely should advanced technology be passed around? Even a
democracy, the author asserts, must restrict its dissemination when national security or
commercial value is at stake and prohibit access to potential enemies or competitors. Does
that seem to contradict the last sentence—that free exchange of scientific information is
required? No: There is a scope difference between scientific information in general, which
the author argues should be kept free, and advanced technology, which may be restricted.

Now, all of that depends on the ability to make a distinction between that which should be
freely exchanged and that which may be prohibited, so (D) is an assumption that the author
must make: There has to be a way to separate out the sensitive tech stuff. By the same token,
if potential enemies and competitors are to be frozen out, it has to be possible to identify the
potential foes . . . so (A), too, is an assumption necessary to the logic. If either (A) or (D) is
in doubt (here’s the Kaplan Denial Test at work), then one could not follow the author’s
dictates with confidence.

Given their references to things like public policy and multinational corporations, the
other two necessary assumptions (a/k/a wrong choices) might not seem within the author’s
scope, but they sure prove to be so. The author must rely on (B), that the people in a
democracy don’t need to know all about its advanced technology in order for the country
to function. Why? Because if (B) is false—if the people do need that knowledge—then the
restrictions on disseminating advanced technology are utterly inappropriate. Similarly, if
(E) is false—if one cannot keep advanced technology from crossing borders more or less
willy-nilly—then the whole idea of keeping such technology out of the hands of selected
countries is a ludicrous one. The author must believe, with (E), that multinational
businesses notwithstanding, a nation can restrict where its advanced technology is shared.

All of the above is moot, of course, if you recognize that (C)’s issue—namely, which type of
political system has more advanced technology—is totally outside the scope of the
author’s recommendations, which inferably apply to all countries, “even in a democracy.”
(That’s what “even” means in this context: The advice is not just restricted to democracies.)
With luck you saw (C)’s irrelevant distinction before spending an unwarranted amount of
time on the other four choices.

• When a stem indicates that the right answer is an “odd man out,” consider looking
for it directly as your first line of attack. Slogging through the four wrong choices
may take more time. Don’t do so unless you have to.

11. (B)
The question wants an inference about typical ice ages, which are mentioned only in the
last sentence, so start there. During an ice age, “however,” precipitation is trapped as ice on
ice caps. What’s that “however” in contrast to?. It’s the previous sentence, which asserts that
“this phenomenon” usually has no effect on the ocean’s composition. Therefore, we can be
sure that the trapping of precip on ice caps during an ice age does have an effect on the
ocean’s composition. See how that works? Go back over that again—follow the logic. Now,
to understand the effect, we have to understand the cause, the “phenomenon.” To what
phenomenon are they referring?

© K A PL A N 51
LSAT PREP ________________________________________________________________ LSAT Test I Explained: Section III

It’s in sentence one. Water vapor evaporates from the oceans with more oxygen-16 and less
oxygen-18 than seawater contains. And that should allow you to follow the steps through.
If all of that water vapor returned to the oceans as precipitation, then there’d be no
problem; the proportions of oxygen-16 and -18 would remain pretty much stable, just as
sentence two says they do. But if during an ice age, the water vapor doesn’t return to the
oceans—if it’s captured by ice—what’s going to happen to the oceans? Seawater is going to
have less oxygen-16 (because that water vapor not returning to the oceans is oxygen-16-
rich), and will have more oxygen-18 (because water vapor is oxygen-18-poor). That second
outcome is choice (B).

(A) contradicts the text which says water vapor and seawater have manifestly different
proportions of oxygen -16 and -18.

(C) Even setting aside the fact that the term “interglacial periods” never appears in the
stimulus—and hence is a poor prospect for an LSAT inference—(C) has it all bollixed up.
The issue is how much oxygen-16 and -18 are returned to the oceans, not how much
evaporates from them. There’s no indication that an ice age has any particular impact on the
composition of the evaporated water vapor.

(D) contains an irrelevant comparison, and in any case precip over land is outside the
scope. All of the author’s interests lie at sea. Me hearties.

(E) may be tempting if you start picturing those icecaps melting slowly. But the author’s
sole interest is in comparing the composition of normal seawater and ice-age seawater, and
the rate of change is never taken up. For all we know, seawater’s composition changes at the
very same rate whether there’s an ice age or no.

• Don’t just rush through your reading of each question stem. Milk it for all possible
help, all possible hints. Here, it’s not enough to recognize Q. 11 as an inference
question—you’re told it’s an inference about ice ages, which directs your attention to
the last sentence as the most important one. You’ll waste less time, and get less
distracted, if you let the question stems help you out.

• Stick closely to the topic and scope of the stimulus when seeking an inference. The
further a choice departs from that which the author takes up, the more likely it is that
that choice is dead wrong.

12. (D)
The stem alerts you in advance that the reasoning is flawed, but given this argument,
hopefully you’d recognize the presence of weakness—and the nature of that weakness—
even without a hint. Why should the drop in household savings become more pronounced
when the population gets older, just because “older people have fewer reasons to save”? If
those reasons, although fewer, are stronger than young people’s reasons, then by the
author’s own logic we should expect to see older people saving more money than kids do.
And that’s what (D) points out. The number of reasons people have to save money has little
or nothing to do with the nature of those reasons, and hence nothing to do with who is more
likely to save.

52 © K A PL A N
LSAT PREP ________________________________________________________________ LSAT Test I Explained: Section III

(A), while true—the author does fail to do those things— fails to take up the older vs.
younger contrast that’s at the heart of the argument. What’s truly missing is what (D) is
getting at: the relative strength and weakness of each group’s reasons for saving.

(B) The argument does nothing of the kind. This “negative savings rate” concept is totally
irrelevant to the author’s prediction about whether older people will continue to save less
and less money.

(C) The argument need not cite evidence as to increasing age, because the author’s
prediction is hypothetical: “If the average age...continues to rise....” That hypothetical
prediction doesn’t hinge on proving that the average will in fact increase.

(E) brings up an irrelevant distinction, between pre- and post-tax income. That has nothing
to do with the strength or weakness of the prediction in question.

• When you’re flat-out told that an argument is faulty, endeavor to articulate to


yourself just what that flaw is, prior to looking at the choices. Those extra seconds of
evaluation can really pay off in more right answers and less wasted time.

13. (C)
You can tell that the point of the argument comes in the first clause—The pit bull is not a
breed—because everything that follows it is signaled by “as do...” and “It is like...,” both of
which are reliable signals of evidence, examples, and analogies. Both police dogs and
Seeing-Eye dogs—terms that are “like” the term “pit bull,” remember—are of the German
shepherd breed, we’re told; what differentiates them is what they do. Inferably, then, a “pit
bull” could be a dog of any number of breeds, because it’s a designation of what the dog
does. (C) picks up on this analogy: If “Seeing-eye dog” and “police dog” designate dogs
according to what they do, and “pit bull” is like those terms in this respect, then a dog can
be designated a “pit bull” based only on what the dog does, irrespective of its breed.

(A) is quite possible, logically speaking, since “pit bull” is a term for a function not a
breed. But it’s hardly the point the author is leading toward. The role of the German
shepherd here is to illustrate the difference between breed and function.

(B) Au contraire, a pit bull is distinguished by what it does and not by appearance alone.

(D) repeats evidence. The phrase “It is like” introduces an analogy, and the last sentence
(which is (D), after all) is there to explain what that analogy is all about.

(E) Distinguishing between breeds is totally outside the author’s scope.

• Questions like this one are made much easier if you’re in the habit of reading
arguments for their structure. Whenever you read a sentence, ask yourself, “Is it the
conclusion? Is it all or part of the evidence? Is it background info, or filler? Or a
restatement of something else in the argument?” And speaking of structure . . .

© K A PL A N 53
LSAT PREP ________________________________________________________________ LSAT Test I Explained: Section III

• Use Keywords. Recognize and interpret the signals that authors use to organize their
prose. Since so many questions hinge on structure, Keywords are reliable signposts
for saving time and racking up quick and easy points.

14. (E)
If, as sentence one says, a marketplace is a necessary condition of a monetary system, and if,
as sentence two says, fourth century Mesopotamia lacked marketplaces, then clearly
Mesopotamia during that period lacked a monetary system. And that’s (E), plain and
simple. Frankly, a student reading the first two sentences and putting them together might
have come up with the proper inference on her own, sought it among the choices, and
fixed on (E) in record time. . . and if that describes the method that you employed here,
well, good work!

(A) The argument leaves open the possibility that other fourth century B.C. populations
besides Greek cities had monetary systems. Not Mesopotamia, though.

(B) is an improper inference of causation. In fact, since a marketplace is a necessary


condition for a monetary system, it’s impossible to have a monetary system before the
development of a marketplace.

(C) We’re told that both Greece and Mesopotamia traded. With each other? Who knows?

(D) Mesopotamia’s situation post-fourth-century-B.C. is outside the scope. We can’t infer


anything about what developed later on.

• Most of the time, the right answer to an inference question will hinge on translating
one sentence of the stimulus, or (as here) combining two sentences. Rarely if ever do
you have to combine all of the data to arrive at your answer. Assume that you can get
away with much less work than that! (See Q. 16, below.)

15. (B)
Businesses should introduce some variety into their software systems (that’s the
conclusion, signaled by “therefore”), because using only one system leaves a company
intensely vulnerable to vandalism. That logic, however, relies on the assumption that
letting a virus loose in a system is worse than rebuilding a system post virus. (B) asserts that
that assumption is true, and hence it supports the reasoning. Use the Denial Test to check
it: If (B) is false—if it’s no more expensive to rebuild a system than to prevent a problem—
then the wisdom of the author’s advice is seriously in doubt.

(A) is an irrelevant generalization about standardization and its effect on business.

(C) Since the argument is solely concerned with businesses that have compatible systems,
(C) is beyond the scope.

(D) The argument is about the damage caused by vandals, not about the inherent
destructiveness of programs. And (D)’s other topic, non-linked computers, is equally
outside the scope.

54 © K A PL A N
LSAT PREP ________________________________________________________________ LSAT Test I Explained: Section III

(E) is true. But businesses that do need to share data are the topic here, because they’re the
ones at risk. (E) neither supports nor weakens the logic.

• The conclusion is what every argument is based on. So if a Keyword signal like
“therefore” jumps out at you, pay special attention to what follows. In some cases,
you can even jump right to the “therefore” in order to read and paraphrase the
conclusion first, and then go on to see how the author reached it.

16. (C)
The right answer, (C), is a paraphrase of the stimulus’ fourth sentence: The variations that
the author is touting will “virtually” eliminate problems, but not necessarily solve ‘em all.

(A) demonstrates a classic fallacy: the fallacy of denying the antecedent. This argument is
saying that if variety is introduced, data can be protected. We cannot deduce from that that
should variety not be introduced, data will be lost. Not necessarily!

(B) According to sentence three of the stimulus, a virus-wielding vandal can destroy
“much” of a company’s data. (B) goes too far.

(D) is outside the scope. The topic is businesses that have multiple-linked systems.

(E) comes off sounding more like an au contraire choice rather than a valid inference.
Contrary to (E)’s spirit, the author is touting minor software variations as a way to make it
tougher to access data—at least for vandals, anyway.

• Keep practicing with, and mastering, the logic of if/then statements. There’s no
single topic that has as much impact on your LSAT score.

17. (D)
Superior conductors (“super-conductors”?) can inspire even top orchestras to work
harder, according to the author, but such authority cannot be claimed, it must be earned.
How does the conductor earn such authority? Simple—by virtue of his or her artistic
interpretation. If the orchestra respects that interpretation, evidently the musicians will
grant the conductor the authority to intensify the rehearsals in order to realize this vision.
But wait a minute . . . What if the artistic interpretation the conductor is “pursuing”
remains ambiguous to the musicians of the orchestra until it’s fully realized? Doesn’t this
short circuit the author’s logic? It sure does: How can the conductor win authority based
on an interpretation that hasn’t yet been realized? In order for this argument to work, the
author must presuppose that top orchestras can appreciate an interpretation before
bringing it to realization.

(A) For all we know, a superior conductor can use the same interpretation for a
composition with many different orchestras and still inspire each orchestra to work harder.
The argument is not damaged by denying (A), so (A) need not be assumed.

© K A PL A N 55
LSAT PREP ________________________________________________________________ LSAT Test I Explained: Section III

(B) “Never satisfied” is a little strong, don’t you think? There’s no reason why a superior
conductor couldn’t be well satisfied by a performance by a top orchestra, which means that
(B) is not necessary to this argument.

(C) goes against the grain of the argument: At least in some cases it takes hard-won
authority on the part of a conductor to inspire an orchestra to put in additional work.

(E) is out of the scope. The argument concerns the characteristics of the superior
conductor, which is a different issue from whether top orchestras are always led by those
conductors. For all we know, some top orchestras are led by second-rate conductors.
(Would they be “semi-conductors”?)

• Always pay attention to the scope of the argument, and recognize when a choice
subtly brings in some element outside of this scope. The author doesn’t make any
claims about whether top orchestras in practice (for instance) are always led by first-
rate conductors, so (E) strays from the argument and thus need not be assumed.

• Don’t be fooled when the testmakers turn the tables on you in the choices; that is,
they switch around some element of the argument and present this new concept in a
choice. Here, the passage involves the orchestra’s assessment of the conductor’s
performance. (B), on the other hand, centers on the conductor’s assessment of the
orchestra. This inverts the situation in the stimulus, and there’s no way such an
inversion need be assumed (that is, presupposed) by the argument.

18. (E)
The stem tells us there’s a discrepancy at work in this one, so immediately we should be on
the alert for something that doesn’t seem to add up. Hopefully, you had no problem
spotting the supposed inconsistency detailed in the short stimulus. Pared down, here’s the
strange situation: The amount of oil in U.S. proven oil reserves is the same as ten years ago,
but oil consumption has increased. One possible explanation is ruled out—no new oil
fields have been discovered. So where’s the extra oil coming from? (E), if true, solves the
mystery: It’s coming from the same proven oil reserves. Thanks to technological advances,
these oil fields can now turn out more oil, because some oil previously thought to be
unextractable is now considered extractable. “Proven oil reserves” are explicitly defined as
the amount of oil “considered extractable from known fields.” “Considered” here is the key:
If this consideration has changed in the way noted in (E), then it’s no longer surprising that
the proven oil reserves are at the same level as ten years ago even though consumption
continues to increase. (E) thus reconciles the apparent discrepancy.

(A) presents an irrelevant comparison. Since the passage—and the supposed discrepancy
contained therein—is based exclusively on the level and consumption of domestic oil, any
comparison of domestic oil to imported oil has no bearing on the argument.

56 © K A PL A N
LSAT PREP ________________________________________________________________ LSAT Test I Explained: Section III

(B) So what if the rate of growth is declining? The fact that consumption is still growing
(albeit at a slower rate) preserves the mystery. If consumption is growing at all (no matter
how slowly it’s growing compared to ten years ago), and if there are no new wells, how is it
that the oil level is the same as ten years ago? (B) doesn’t help us resolve the discrepancy.

(C) The only possible thing (C) helps explain is why there have been no new oil fields
discovered recently. As for the seeming discrepancy at hand, no help.

(D) Price? Like (C), (D) offers a possible explanation for a single element of the argument,
in this case, the increased annual consumption of oil. The issue here is the amount, or level,
of oil, and a decrease in price by itself, while possibly a factor in the increased
consumption (maybe the decreased price has driven the increased consumption), still has
no effect on the surprising amount of oil in the U.S. reserves.

• To resolve a seeming discrepancy or apparent paradox, the correct choice must hit at
the central element of the unusual or surprising finding or result. Here, the concept
of “amount” is at the heart of the mystery: Given the circumstances, how could there
be the same level of oil now as there was ten years ago? (E) hits at that issue by
strongly suggesting that a greater amount of oil from the proven wells now fits the
definition in the first sentence. Conversely, choices focusing on other kinds of oil
(A) and price (D) can’t help us, and should have been easy choices to axe.

• As mentioned above, (C) and (D) offer possible explanations for certain facts in the
stimulus. But that’s not enough; we’re looking for a choice that resolves the
discrepancy. Don’t get sucked into a choice just because it’s relevant to something—
the correct choice has to be relevant to the task stated in the stem, which in this case
is to reconcile the apparent discrepancy.

• Definitions are key. If the testmakers go out of their way to define a phrase or term,
make sure you keep that definition in mind. Often, noticing such a definition and
taking account of it accordingly will be the key to getting the point. Sure, this seems
like a strange situation—U.S. oil is at the same level as it was ten years ago while
demand and consumption have increased—but when we focus on the fact that oil
level in this case is strictly defined as the amount considered extractable, it becomes
easier to see how (E) helps explain away the seeming discrepancy.

© K A PL A N 57
LSAT PREP ________________________________________________________________ LSAT Test I Explained: Section III

19. (C)
Maybe you saw the problem with this logic right away: The evidence deals with train
service and customer satisfaction, while the conclusion jumps to the concept of railroads as
“a successful business.” Are these things the same? No—you probably know the style of
the testmakers better by now. As you’ve most likely noticed, they’re very particular about
the way they use words on the LSAT. Here, the scope shifts from the first sentence to the
last. Even though “train service” and “successful business” seem correlated in some way,
they’re not the same thing. The author should have explicitly stated that there’s a
connection between the two; without such a stated connection, the argument falls flat. It
seems like an obvious connection, but it’s nonetheless necessary for the argument to work.
By relating service and customer satisfaction to the success of the railroad business, choice
(C) bridges this narrow gap in the argument. (C) is thus the assumption on which the
argument relies.

(A) For all we know, these two types of train service may be closely related. (A) seems to
lend support to the assertion in the first sentence, but why the railroads can’t run both
services at once successfully isn’t discussed and isn’t relevant to the argument as a whole,
so (A) need not be assumed.

(B) Prioritization isn’t an issue. All the author does is posit a condition that’s necessary if a
railroad is to be a successful business. Thus, the author need not assume that being a
successful business is the railroad’s first priority; this argument could work even if being a
successful business was far down on the list of a railroad’s priorities.

(D), far from a necessary assumption, is an unwarranted inference based on the final
sentence. The last sentence expresses the concept of necessity (it’s necessary to concentrate
on one service to be successful), while (D) interprets this as a statement of sufficiency (all the
railroad needs to do is concentrate on one service to be successful).

(E) goes astray by trying to mix the two service lines, but commuters’ demand for freight
service is an irrelevant issue that in no way plugs the hole in this argument.

• Scope shifts cover a lot of ground. Here’s an example where the author shifts the
scope of the argument by introducing a new word or phrase, in this case, “successful
business.” If you noticed the shift from “train service” in the evidence to “successful
business” in the conclusion, you probably had no problem picking out the
assumption in (C) that bridges the gap between these two distinct elements of the
argument.

• Don’t do the author’s work for her! You may have missed the distinction above if
you inferred the connection in choice (C) on your own. Sure, it’s common sense that
if you don’t serve customers well, you probably won’t have a successful business.
Regardless, making this judgment still entails taking a logical leap, because this
author never makes this connection that holds the argument together. It’s the
author’s job to present as tight an argument as possible, and that includes tying up
the loose ends. If (C) weren’t true (the Denial Test)—and it’s possible theoretically
for railroads to succeed without serving customers particularly well—then this
argument would fall apart.

58 © K A PL A N
LSAT PREP ________________________________________________________________ LSAT Test I Explained: Section III

20. (E)
We need to find an inferable hypothesis based on the two-sentence stimulus. We get two
ideas in the first sentence: Most U.S. people don’t regard big or small business as
particularly efficient or dynamic, although they do seem to think both forms of business
offer fairly priced goods and services. However (and this Keyword should always jump off
the page at you), we’re told small business is generally perceived as more socially
responsible (“a force for good in society”), while big business, according to most people, is
only socially responsible in times of prosperity. Some inferences are difficult to pre-
phrase; you may have found that evaluating the choices was the way to go here. Most of the
wrong choices touch on issues that fall outside of this very narrow scope. The correct
inference is found way down in (E), and comes from combining the last part of sentence 1
with the last part of sentence 2. There are times (non-prosperous periods) when most
people believe that big business is not socially responsible even though most people
regard big business as providing consumers with fairly priced goods and services. Why
not? As (E) puts it, there must be something more to the “social responsibility” of big
business than simply providing consumers with fairly priced goods and services.

(A) goes against the spirit of the passage, which suggests that most people in the U.S. do
have definite views on, and thus give thought to, the value of business to society.

(B) and (C) We have no idea how the negatively perceived “efficiency/dynamism” factors
from the first sentence relate to most people’s attitudes regarding the other elements
discussed (price, social responsibility), so we can’t infer what would result from changes in
business’ efficiency or dynamism or changes in the public’s perception of same.

(D) We know that big business is seen as socially responsible in times of prosperity, but we
don’t know why. This perception may be dependent on the perception that big business
provides consumers with fairly priced goods and services, but this perception might be
caused by something else. So we can’t tell what would happen if big business didn’t
provide consumers with value for their money, and we can’t infer (D).

• Formal logic sneaks around the LSAT in subtle ways. Notice that the correct
inference required a translation of the last sentence, based on an understanding of
the formal logic term “only”: “Perceived as responsible ONLY in times of prosperity”
means “NOT perceived as responsible in times of NON-prosperity” (economic
downturns, recessions, depressions, etc.). Make sure you’re adept at interpreting the
formal logic terms (only, some, always, most, all, none, and so on) that appear
throughout both the Logical Reasoning and the Logic Games sections.

• The correct answer to an Inference question is a statement that is certainly true (or is
virtually so) based on the stimulus. However, the correct answer need not combine
all or even most of the statements in the stimulus. For this reason, it can be difficult
to form a pre-phrase to Inference questions. So don’t worry if you don’t have a
strong idea of what the correct answer should look like before you hit the choices.
Just keep a strong grip on the scope of the argument, and look to eliminate choices
that depart from that scope. (See the next bullet point.)

© K A PL A N 59
LSAT PREP ________________________________________________________________ LSAT Test I Explained: Section III

• Eliminating choices in Inference questions is easier if you can recognize what you
don’t know from a given situation. Here, we’re given a host of conditions and
perceptions. What happens when these change? “If big business were MORE
efficient (B)”; or “if small business were MORE dynamic (C)”, we don’t know.
Choices like (B) and (C) should be crossed off as soon as you realize that they
discuss issues that go beyond those discussed in the passage.

21. (C)
We’re looking for an assumption in an explanation, so your first task is to locate the
explanation and the observation it is supposed to explain. The Keyword phrase “This is the
reason that . . .” gives the structure away: The first sentence is meant to explain the
observation discussed in the second sentence. Squirrels can run up steep inclines very
quickly whereas large animals slow down while moving uphill. What is it about squirrels
that gives them the edge? The explanation claims that the energy required to run uphill is
proportional to body weight, but the energy available to run uphill is proportional to
surface area. So the animals that have an easier time running uphill have relatively more
surface area as compared to their body weight. How does this apply to the observation
about squirrels? We know that squirrels weigh less than larger animals, and so they don’t
need as much energy to run up hills, but we can also infer that squirrels have less surface
area than larger animals, and so they have less energy available to do the job. So while
squirrels have an advantage in one aspect (body weight), they have a disadvantage in
another (surface area). So what else has to be true about squirrels? As (C) puts it, squirrels
must have a high surface area to weight ratio, which means that they have more surface
area relative to their body weight. (C) fills in the gap by explaining why the squirrels’
disadvantage in surface area is more than compensated by their advantage in body weight.
Squirrels may have a little less energy to do the job, but this is more than made up by the
fact that larger animals are much, much heavier.

(A) Au-contraire: In the event that large animals weigh more than small animals (which is
most likely the case), the first line of the stimulus supports the opposite of (A).

(B) The issue here is moving uphill. The explanation therefore need not rely on any
comparison of the general speed of small and large animals. Moreover, (B) ignores the surface
area issue and thus fails to tie together the theory and the phenomenon it’s meant to explain.

(D) and (E) both focus on ratios that are never mentioned in the stimulus (energy output is
proportional to surface area; energy needed is proportional to weight), so there’s no way
that either of these can be the assumption on which the explanation depends. Moreover, (E)
has the same problem as (A)— “run” is simply too general because the stimulus focuses on
moving uphill.

• Keep the terms of the argument straight: Even though the argument focuses on the
concept of “energy,” it’s crucial that you recognize that two different kinds of energy
are discussed, and that you keep the distinction between them in mind. (You may have
benefited by jotting down the words “needed to go up—weight” and “available to go
up—area” in the margin to represent the two kinds of energy and their respective
sources. It’s great if you were able to keep this distinction clear in your mind, but don’t
underestimate the power of shorthand to help even here on the Logical Reasoning section.)

60 © K A PL A N
LSAT PREP ________________________________________________________________ LSAT Test I Explained: Section III

• Visualize in order to make the passage your own. Why not picture a squirrel running
up a tree while an elephant lumbers up a nearby hill? Maybe such a mental picture
would help to solidify in your mind the concept of the ratio between surface area and
weight and the consequences of such a ratio on an animal’s efforts to move uphill.

• This question is not a bad question to skip, especially if you were running low on
time and noticed the two-question stimulus, Qs. 24-25, at the end of the section. Two
questions for the price of one in 24-25 may be preferable to suffering through the
somewhat complex ratios and comparisons here.

22. (E)
In trying to absorb the 1980’s brand of selfishness into the broader context of human history,
the author employs two radically different uses of the word “selfish,” as (E) correctly
points out. “Selfish individualism” is certainly different from “selfish concern for the human
species”, but the author lumps them together. To show that the 80’s have gotten a bum rap,
the author would have to show that the individual kind of selfishness that characterized the
80’s has been evident throughout history. Evidence regarding the different form of
selfishness described in the last sentence is no help at all, which is why the argument falls
flat.

(A) The claim that selfishness has been present throughout history certainly is relevant to
the argument; it’s simply not backed up correctly. As we’ve seen, it’s the form of historical
selfishness the author discusses (selfish concern for the human species) that gets him into
trouble. As discussed above, the argument would work nicely if the author offered
historical evidence regarding the individual kind of selfishness characteristic of the 1980’s.

(B) No such evidence is necessary—the argument is not predicated on the belief nor is it
intended to show that humans act selfishly more often than not. In fact, it boggles the mind
to think of a way in which this even could be statistically determined.

(C) Au-contraire—the author states that selfishness has been present throughout history.

(D) The classic outside the scope choice: What do other species have to do with this
argument? Nothing—this is about humans and humans only, so the author need not
consider the behavior of other species.

• Equivocation is the fancy name for a situation in which different meanings of the
same word are used within an argument. Usually, the resulting argument is a flop.
You need not know the formal name for this device, but you must recognize this
special form of scope shift when it appears on the test.

• In Logical Flaw questions, many of the wrong choices focus on omissions—things


the author hasn’t done. Don’t choose one of these choices simply because it may be
true. The test is not “did the author leave this out?” but rather “did the author leave
this out and should he or she have put it in?” Don’t fault an author for leaving out
something he or she is not obligated to include as part of the argument. Here, (B) and
(D) fit this description perfectly, and hopefully you were able to axe these choices
quickly.

© K A PL A N 61
LSAT PREP ________________________________________________________________ LSAT Test I Explained: Section III

23. (C)
We’re given the results of a survey that are used to support a journal’s decision, and we’re
asked to find the choice that provides the best evidence that the decision was the correct
one. Many strengthening questions involve shoring up a central assumption or recognizing
and discounting a potential flaw in the logic. And maybe you saw the problem right off the
bat: “Sixty-two percent of those that returned the questionnaire supported that change.” “Of
those that returned the questionnaire” should have sent up a red flag: Maybe the people
who bothered to return the questionnaire are more likely to agree with the changes; maybe
those who weren’t thrilled with the proposed change were less likely to reply.
Furthermore, maybe the sample size was very small, and thus not a good indicator of the
preferences of the larger potential readership. Any of these problems may very well skew
the results, and would make us less likely to believe that the change will lead to an increase
in readership. However, if (C) were true, and the opinions of the entire potential
readership matched almost exactly the opinions of those that returned the questionnaires,
then close to sixty-two percent of the potential market favors the change. In that case,
changing to the new format would be more likely to increase readership, and thus (C)
would provide evidence that the journal’s decision will have the desired effect.

(A) still doesn’t discount the possibility of a non-representative sample: It could be true
that the questionnaire was sent only to those who for some reason or another are likely to
support the change. It also leaves open the possibility of a very small sample size, one that
may not reflect the attitudes of the journal’s larger potential readership.

(B) So? How do we know these changes were successful?

(D) Cost is not the issue—we’re interested solely in whether the change will have the
desired effect, which is increased readership. (Even if this cost cut were passed on to the
reader, which theoretically could increase readership, the journal’s decision is based on the
general positive reaction to format changes, and thus the argument needs to be strengthened
or weakened on these grounds, not on grounds irrelevant to the survey such as a price
change.)

(E) could only weaken the argument. If the readers who disliked the old format were more
likely to respond to the questionnaire than those that liked the old format, then the survey
results would seem to be biased against the old format, which suggests that changing it
may not have been such a good idea after all.

• The sample of people questioned is always relevant when evaluating arguments


based on surveys or polls. One of the most common logical flaws that appears in
arguments containing surveys or polls is the non-representative sample—cases
where the results cannot necessarily be trusted because the group of people who
generated the results aren’t representative of the population in general, or at least the
population on which the surveyors suggest the survey or poll was based.

• Recognizing an argument’s central assumption will sometimes lead you directly to


the answer, or, other times, may get you there indirectly. Here, answer choice (C)
shores up this potential problem and thus supports the journal’s decision by
ensuring that the same positive feedback would be characteristic of the sample
group relevant to the journal’s objective.

62 © K A PL A N
LSAT PREP ________________________________________________________________ LSAT Test I Explained: Section III

• Don’t let wordy question stems throw you; translate them into something that’s more
user-friendly. Here, we’re essentially asked to strengthen an argument, only in this
case the argument is phrased in terms of a journal’s decision based on a
questionnaire survey. The survey functions as the evidence, and the decision is like
the journal’s conclusion.

24. (E)
We’re looking for a principle with which to defend Shanna’s position against Jorge’s
rebuttal, so naturally, the first step is to understand their respective positions. The topic is
art: Shanna believes that the owner of a piece of art has the ethical right to destroy that
work if he or she doesn’t like it or if it becomes inconvenient to care for. Jorge narrows the
scope to unique works of art, which he believes carry an additional burden—an owner
should not destroy a unique work of art with aesthetic or historical value because such a
work belongs to posterity and therefore must be preserved. It’s possible to pre-phrase the
kind of principle that will favor Shanna’s position over Jorge’s. It must somehow reflect the
notion that the rights of the individual owner supersede the requirements of posterity, or
at least something along those lines. (E), which grants such power to the individual, is a
principle that reflects this notion perfectly. The general nature of the principle— “do what
they wish with what is theirs”—covers Shanna’s position entirely; certainly even unique art
works with aesthetic or historical value (the kind Jorge wishes to protect) fall into the
category described in this choice. (E) thus supports Shanna’s position against Jorge’s, and
gets the point for 24.

(A) Jorge posits an obligation for all owners of unique and important works of art. A
principle that covers only the “serious student of the history of art” is too narrow and weak
to help Shanna reply to Jorge’s mandate. (A) also leaves out the “inconvenience” issue. The
serious students in this principle may still want to destroy a work of art because it has
become inconvenient to care for, but Jorge’s position would still effectively forbid this,
regardless of whether the principle in (A) were accepted.

(B) Au-contraire: This supports Jorge’s position over Shanna’s.

(C)’s not be as strong as (B), but it too works against Shanna’s position, offering another
reason why it may not be a good idea to allow art owners free reign to destroy their art.

(D) is mostly irrelevant and could only support Jorge’s position. We can’t tell whether
works of art would qualify as entities over which no one could rightfully claim absolute
rights of ownership, but if works of art did so qualify, then there would be limits to
ownership rights, which could prohibit the kind of destruction that Shanna finds
permissible.

• This is an offshoot of a Strengthen/Weaken question. At the same time, the correct


principle must strengthen Shanna’s argument while weakening Jorge’s. All
Strengthen/Weaken questions include the phrase “if true” or “if accepted” in the
stem; this tells you not to argue with the answer choices or question the likelihood of
each, but rather to accept each choice as true and concentrate instead on what effect it
has on the argument or arguments in the stimulus.

© K A PL A N 63
LSAT PREP ________________________________________________________________ LSAT Test I Explained: Section III

• Pay attention to the tone of the choices—notice how strongly worded the right
answer is: The rights of individuals with regard to their property “must not be
compromised . . .” There’s nothing wishy washy about that, and indeed (E) leaves
Jorge no recourse—the obligation he discusses goes out the window if the principle
in (E) is accepted.

25. (A)
In simpler terms, the question stem asks us to find the statement amongst the choices that
one person in the dialogue would consider true and that the other person would consider
false. Understanding what the testmakers are after here helps us to conceive of a logical test
for each choice. Take (A), for example. What would Shanna think of the truth of this
statement? She would think it was true, right? Sure—if you don’t like it, according to
Shanna, you’re ethically justified, “for that reason alone,” to destroy it. Would Jorge have
an opinion regarding the truth of the statement in (A)? Yes—he would not believe the
owner of such a portrait can ethically destroy it “for that reason alone.” Based on his
argument, he would impose another restriction: The painting must not be “a unique work
of art with aesthetic or moral value.” If it is, then it should not be destroyed. Shanna would
agree with the truth of (A), while Jorge would not, which means that (A) is the answer we
seek.

(B) Public viewing is beyond the scope of both arguments. The issue here is the right to
destroy versus the obligation to preserve.

(C) and (E) are out of the scope. The issue here is whether and under what circumstances it
is morally permissible to destroy works of art. Whether intentional damage and destruction
happens often or not, (C), is irrelevant to this question. As for (E), what does legal
permissibility have to do with moral permissibility?

(D) Shanna would agree with this statement even if the piece of sculpture were unique. We
can infer that Jorge would agree with this statement as well; his mandate for preservation
applies only to unique works of art. So (D) turns out to be an au-contraire choice, as both
parties would seem to agree, not disagree, about the truth of this statement.

• Mind those scope shifts! After two full questions dealing with this topic, the word
“legally” in choice (E) should stick out like a sore thumb.

• This is an offshoot of a Point-at-Issue question, but unlike a typical Point-at-Issue


question, we’re not asked for the specific idea that comes between Shanna and Jorge.
Rather, we’re asked to use what we know about the discrepancy in their positions to
recognize a statement that one would regard as true and the other would regard as
false. (That is, after all, what “disagreeing about the truth of a statement” means.
Always translate a confusing-sounding or complex stem in order to get it into a form
that’s easier to use.)

• In a double-question stimulus, use your work on the first question to help you
answer the second. You should have had a very clear idea about the debate from
your work on Q. 24, so it shouldn’t have taken long to jump to the choices and settle
on correct choice (A).

64 © K A PL A N
SECTION IV:
LOGICAL REASONING

© K A PL A N 65
LSAT PREP ________________________________________________________________ LSAT Test I Explained: Section IV

1. (C)
The stem immediately alerts us to the two major elements of the stimulus—a problem and
a proposal with which to solve it. It also tells us that the proposal falls short; our job is to
find out why. The problem is the storage of nuclear waste. Currently, waste is stored on-
site, but this won’t do for the long term. Since no storage solution is yet forthcoming, some
people propose that we scrap the entire nuclear system altogether—shut down the present
plants and build no new ones. This is the “solution” referred to in the stem. What’s wrong
with this? It totally ignores a huge part of the problem—the nuclear waste that already exists
and that is stored “on-site.” The storage of existing waste, after all, is the problem that the
solution is supposed to address. (C) picks up on this flaw inherent in the proposal.

(A) First of all, we can’t infer that shutting the plants will “prevent” such development;
maybe it would spur such development on. Secondly, even if we accept the truth of (A), it
doesn’t point out why the proposal falls short of solving “the problem it addresses”—that
is, it ignores the issue of radioactive waste altogether.

(B) falls prey to the same weakness as (A); it doesn’t address why the solution fails in
regard to the original problem, the storage of existing nuclear waste.

(D) is way out there. Not only does it ignore, like (A) and (B), the main issue of current
radioactive waste, but it doesn’t even directly address the proposal or any possible
weakness of it.

(E) is consistent with the evidence in the first sentence—while waste is stored on-site for
now, this is not a good long-range solution, or as (E) puts it, the risks of unsafe disposal lie
in the future. So? All (E) does is reinforce the notion that prompted the proposal—it in no
way explains why the proposal will fail to solve the problem it addresses.

• Scope shifts take many forms; sometimes they’re temporal in nature. Watch out for
instances where authors switch liberally, without warrant, between considerations of
the past, present, and future. Here, the problem is based in the present: What to do
about existing waste currently stored on-site? But the solution shifts the focus to the
future—we’ll simply create no new waste—and in doing so ignores a big part of the
problem it supposedly is intended to solve.

• Why do we at Kaplan suggest that you read the question stem first in Logical
Reasoning? Just look at what we get out of this stem: We’re alerted to the fact that the
passage discusses a problem, and a proposed solution that falls short. This is a lot of
information to have before you get to the passage, and hopefully you used this
information to guide your initial reading of the stimulus.

66 © K A PL A N
LSAT PREP ________________________________________________________________ LSAT Test I Explained: Section IV

2. (A)
Here we have lots of formal logic statements hidden in what appears to be a casual
argument. Working backwards from the last sentence: ALL Hawaiian fruit flies are thought
to be descendent from the same one or two ancestral females. Now look at the sentence
right before this: The picture-winged drosophilid is a subset of Hawaiian fruit flies. That
means one and only one thing: All Hawaiian picture-winged drosophilids must therefore
have the same characteristic of the entire group of Hawaiian fruit flies—they must all be
believed to be descendent from the same one or two ancestral females, choice (A).

(B) We have no way to infer that picture-winged drosophilids are unique to Hawaii; just
because this species is found there doesn’t prevent it from appearing elsewhere.

(C) distorts the argument by attempting to forge a connection between the first and last
sentences. However, no such connection is indicated, because the scope of the first sentence
is “worldwide” while the last sentence strictly concerns Hawaii. Specifically, it could very
well be that NOT all, or even very few of the 1000 - 2000 worldwide species of fruit flies
share the Hawaiian fruit flies’ characteristic descent from one or two ancestral females.

(D) takes its thrust from the idea in the second sentence, that Hawaii has the most diverse
population of fruit flies in the world. Even if 500 new fruit fly species were discovered,
how would we know whether Hawaii would lose its fruit fly diversity crown? There’s no
way to tell other than by pure conjecture, and we’re in no position for that. (D) is not
inferable.

(E) Again, like in (C), we get a connection between Hawaii and the world that is in no way
supported by the stimulus. While (E) certainly could be true, there’s simply no way to
logically infer it from the limited and focused information in the stimulus.

• Get past the wordy and cumbersome terminology to see the real relationship among
the groups in the stimulus. Here, this situation boils down to nothing more
complicated than this: ALL X (picture-winged drosophilids) are Y (Hawaiian fruit
flies). ALL Y (Hawaiian fruit flies) are Z (thought to be descendent from the same one
or two ancestral females). Therefore, ALL X (picture-winged drosophilids) are Z
(thought to be descendent from the same one or two ancestral females).

• When you’re asked for an inference based on a formal logic stimulus, don’t be
surprised if the answer comes from one sentence or a combination of two sentences.
Here, the first two sentences are totally unnecessary to derive correct choice (A). They
do, however, serve as fodder for the wrong choices.

© K A PL A N 67
LSAT PREP ________________________________________________________________ LSAT Test I Explained: Section IV

3. (C)
Does a feather a bird make? Evidently, the author never considers this question—she takes
it for granted that feather = bird. It may seem so obvious to us that we don’t even realize
that the author fails to make the connection between the feather impression (the evidence)
and the conclusion that this feather impression signifies the earliest bird. What if, during
this era, other creatures had such feathers as the one found in the limestone slab? Does the
conclusion still hold? Of course not—the feather need not signify the earliest bird found to
date, or any bird, for that matter, if it could have come from a different kind of creature.
The author must assume (C)—that NO other creatures of this period had such feathers—in
order for her conclusion to stand.

(A) The relation of the feather to the pterosaurs is of no import here—the pterosaurs are
merely mentioned to lend historical context to the story, but don’t really play any major
logical role in the argument. For the argument to work, the author need not assume such a
connection between the feather and the pterosaurs.

(B) Why would the author necessarily assume anything about earlier birds preceding the
bird in question when she argues that this bird may be the earliest bird?

(D), like (B), goes against the spirit of the passage; the author clearly believes that the
feather belonged to some sort of bird.

(E) Awkwardness? If anything, the description of the feather as “fully aerodynamic”


suggests the author may believe the opposite to be the case.

• The Kaplan Denial Test is simply a matter of common sense. Did you notice how we
employed the Denial Test in the explanation of correct choice (C) without actually
invoking its name? You may have employed this reasoning yourself without even
realizing you were using the Denial Test. If you thought to yourself, “Yeah, if (C)
isn’t true, this whole thing is really shaky”, then that’s exactly what you did. If so,
congrats; the more you internalize this form of reasoning, the better off you’ll be.

• When looking for an author’s assumption, keep your eye out for something the
author takes for granted. “Takes something for granted” is another of way saying
“assumes.”

• The correct choice in an assumption question is the one that contains a necessary
but unstated piece of information. Of the four wrong choices, some may be
irrelevant—things that may or may not be so, but regardless, provide no necessary
support for the argument. Even if a choice seems to be “true,” that’s not enough for it
to be correct; it must also be necessary to the argument to qualify as an assumption.
The last kind of wrong choice is the one that violates the spirit of the passage—one
that seems to suggest the opposite of what the author may believe. Those choices,
like (B, (D), and even (E) here, should be the easiest to eliminate—nothing that goes
against the spirit or tone of the argument can be assumed by the argument.

68 © K A PL A N
LSAT PREP ________________________________________________________________ LSAT Test I Explained: Section IV

4. (C)
The test for the choices is simple. Each choice that supports the conclusion must lend
credence to at least one of the two factors responsible for declining home energy
consumption: reduced standards of living and changes in people’s use of their time. We’re
looking for the exception, the choice that doesn’t support the conclusion—that means the
choice that doesn’t conform to either of these two factors, and (C) is the one: Having cheap
work done to improve home heating systems neither reduces one’s standard of living nor
represents a significant change in the use of one’s time. Essentially, (C) provides a different
reason for the decline, one that might even call into question the researchers’ conclusion
that the factors they cite are responsible for “almost all of the decline.”

(A) represents a change in the families’ use of time; moreover, a restricted use of one’s
house may very well also qualify as a reduction in one’s standard of living. The result of
such action is that less oil is needed to heat the entire house.

(B) mostly supports the “use of time” factor. During the cold months, more people than
before frequent public places, and people tend to spend “longer periods in them.”
Obviously, this represents a change in the way people spend their time, and during this
“outside the house” time, less oil is needed to heat their houses.

(D) is reduced standard of living. They may be colder, but they’re consuming less energy.

(E) hits on both factors. For those who love long hot showers, cutting back may very well
represent a reduction in their standard of living. And in a technical sense, although it may
not be by much, such people are slightly altering the way they spend their time.

• Focus on the task you’re assigned. Here, you’re not looking to support the notion
that home energy consumption has declined—you’re looking for examples that
support the researchers’ conclusion that the decline is due almost entirely to the
factors they cite. If you didn’t read critically to ascertain the true argument here, you
probably thought that all the choices, including (C), supported the conclusion.

5. (B)
Our tricky little senator employs a common political trick: He presents a view that
contrasts that of the administration, but then proceeds to argue as if the administration’s
view doesn’t exist. Even his rhetoric is a carefully constructed attempt to show that only his
party’s view exists or matters: “Hooray for common sense . . .”; “Everyone knows that . . .”
He then concludes that the tax is dead, because no senator would ever vote for something
that will bring about such results—the results that his party predicts but that he now tries
to put over as the only possible interpretation. Through all this, Strongwood is hoping that
people, especially the voting senators, will forget about the other interpretation of the
effects of the tax, namely, the administration’s interpretation. Clearly, in making his
argument, he is assuming that the senators believe the report of his party, not that of the
administration.

(A) Strongwood’s party believes that reducing the tax would increase the deficit. Nowhere
does he or his party suggest what will happen if the tax is increased.

© K A PL A N 69
LSAT PREP ________________________________________________________________ LSAT Test I Explained: Section IV

(C) The “common sense” comment is NOT expressly directed at his opponents. He may
imply by this comment that the administration lacks common sense on this issue, but this is
only a subtle jab, not nearly enough to qualify as “name-calling.”

(D) The notion of “popular” never arises—he simply believes the senators won’t vote for
something that, at least in the view of his party, will have the negative effect of increasing
the deficit.

(E) We don’t know why Strongwood feels his party’s study is superior to that of the
administration’s. As far as we know, he assumes nothing about the relative “objectivity” of
each. He simply announces the superiority of his party’s study, not the reasons for the
superiority.

• Don’t read more into the stimulus than is there. “Hooray for common sense” is much
different from “my dumb-as-dirt thumb-sucking opponents lack the common sense
to see that . . .” The stimulus would have to sound something like the latter
(although probably not as harsh so as to fit LSAT standards) if (C) were meant to be
correct.

6. (D)
Successful economies have been the ones that have included a high level of technological
training. Moreover, we’re told in the first sentence that this will continue to be the case.
Japan is good in this respect, certainly better than Europe, but even Japan has a high
number of people lacking essential skills. What can we infer from this? Certainly (D)
follows logically: Europe needs to get its technical training act together if it wants a more
successful economy. This is borne out by the fact that Europe is lacking in one respect
(technological proficiency) that “will continue to be” needed for a successful economy.

(A) offers an unwarranted comparison that we have no way of corroborating based solely
on the evidence in the passage. Engineers aren’t even mentioned by name, so we certainly
can’t infer a comparison like this.

(B) Even with its flaws, what evidence exists that there’s a better country to serve as the role
model of technological proficiency and training? (B), without support, is not inferable.

(C) We can practically infer that Japan’s economic success is due to a narrow band of
highly skilled labor; the last sentence suggests that since so many of the workers are skilled
in manual labor only, only a relative few are highly skilled in the new technology. But is
this distribution uncommon? Certainly not; this distribution of skilled vs. unskilled labor is
typical in most European countries, and we have no basis to infer it’s much different
anywhere else, since Japan is the cream of the crop in this respect (“the model for this sort
of training effort”).

(E) contains another comparison that has no basis in the facts of the passage. The scope of
the passage includes Europe and Japan, period—we know nothing of the relative
economic success of “most other countries.”

70 © K A PL A N
LSAT PREP ________________________________________________________________ LSAT Test I Explained: Section IV

• Sometimes a small, innocuous looking word or phrase turns out to be the key to the
whole question. “. . . and will continue to be . . .” is such a phrase here. Skim past
such phrases at your own risk.

• Keep the scope of the passage always in the forefront of your mind. In this case,
choices containing words like “engineers” (A) and “other countries” (E) have “scrap
heap” written all over them.

7. (D)
When did the inhabitants of Mexico start playing this ceremonial game? They were
observed playing the game with a rubber ball in 1519, and rubber itself was not used in
Mexico before approximately the year 1000. So the author concludes that the game must
have been invented between approximately 1000 and 1519. Is that right? We know that the
game couldn’t have been played with a rubber ball before that time, but what if the game
had not always been played with a rubber ball (the Denial of (D))? If the game could have
been played with something else (and we’ll leave it to your imagination to determine what
that other object might be), then the game might have been played well before the year
1000, and the author’s argument would fall apart, confirming (D) as a necessary
assumption.

To look at it another way, suppose that someone visited America in 1980 and saw the
inhabitants playing baseball with aluminum bats. If it were established that aluminum bats
were not used before, say, 1960, then would it be fair to say that baseball could not have
been played before 1960? Of course not, since the same game could have been played
earlier with the materials that were then available.

(A) and (C) are out of the scope. The question concerns when the game was originated, and
not the popularity of the game (C) or game-playing in general (A).

(B) If (B) were false, then the window in time in which the game could have been
originated would be narrower, but still within the limits suggested by the author; thus, the
argument would remain intact. Since the denial of (B) does not defeat the argument, (B)
cannot be necessary to the argument.

(E) is irrelevant. Cortez’s observation is important because it shows that the game could not
have originated after 1519. Otherwise, there is nothing special about Cortez; there’s no
reason to require the rubber dating to have been due to him.

• Watch out for scope shifts between the evidence and conclusion! When you spot
them, you’ll be able to form a powerful pre-phrase. In this case, the evidence
concerned an observation of inhabitants playing a game with a rubber ball, but the
conclusion concerned the origin of the game itself. If you asked yourself whether
the same game could have been played with different materials, you were home free.

© K A PL A N 71
LSAT PREP ________________________________________________________________ LSAT Test I Explained: Section IV

• There’s a great deal of synergy between the different Logical Reasoning question
types. For example, many different Logical Reasoning question types require you to
identify assumptions. Question 7 was posed as an Assumption question, but it
could have easily been posed as a Strengthen/Weaken the argument question, or
even a Flaw question. So improving your ability to spot central assumptions will
have many benefits. You’ll do better on Assumption questions, to be sure, but you’ll
also do better on many of the other question types.

8. (C)
The question is whether billboards ought to be torn down. Mary Simms, the rep of the
billboard company, not surprisingly, argues against that notion. She claims that if the
billboards are torn down, “our ability to earn a living will be damaged.” The word “our”
in this case refers to the outdoor advertising company, a fact that has eluded Jack Jordan, a
local merchant. Jack argues that the billboards are “hurting our ability to earn a living.”
Evidently, in his case the term “our” refers to his specific interest group—the town’s
merchants. When he states that the “basis for our business is an attractive community,” he’s
talking about the business community in general—we can tell by the next sentence: Ugly
billboards are a turn-off to potential shoppers. Jordan has misinterpreted the word “our.”
Mary’s claim was not directed towards the business community in general, but towards
her advertising company specifically.

• Many LSAT questions have this format: A claim is made, a response is made, but the
response in ineffective because the responder has made some kind of error, usually
shifting the scope of the argument. So when you see a question in which a claim is
met by a response, scrutinize the response carefully to see if it is on point.

9. (E)
This is a great question to save for last. You have five, count ‘em five, formal logic
statements, and then you have to combine them with a sixth, found in the stimulus. It is
difficult to pre-phrase an answer in this kind of question since the correct answer could
take any of a number of forms, so there’s nothing to do but combine the statements, and
examine the choices one-by-one. To make things worse, the right answer turns out to be (E).
We know that all Capulets are intemperate (E), since all non-Montagues are intemperate
and no Capulet is a Montague.

(A), (B), (C) and (D) all ignore the possibility that there are people who are neither
Capulets nor Montagues, a possibility left open by the rules. Recognizing this possibility
eliminates choice (B) directly. We can deduce that all Capulets are intemperate (which is
correct choice (E)), and they can all be crossed in love, but for all we know there are some
people who are neither Capulets nor Montagues that share these qualities as well, which
axes choices (A) and (D). As for choice (C), we know that all intemperate people are non-
Montagues, but as far as we know they might be immune to love-crossing as long as they
are also not Capulets.

72 © K A PL A N
LSAT PREP ________________________________________________________________ LSAT Test I Explained: Section IV

• Remember, every question is worth the same, but some take more time than others. If
you can tell that a question will suck up too much of your time, leave it for last. If
you don’t recognize this until you are into the question, don’t fight until the bitter
end. Skip it or make an educated guess.

• A valid inference must be true, but need not be based on all, or even most of the
statements in the stimulus.

10. (C)
“Therefore” signals the conclusion: “Somehow,” concludes this author, heavy rains must
promote hurricane formation. How come? Because right after it rains in sub-Saharan
Africa, the U.S. gets hit with hurricanes big time. This logical flaw is classic (meaning that
it goes back to ancient logicians and that it appears frequently on the LSAT). Logicians call
it the “post hoc fallacy,” and though you need not know that name, you’ve got to become
familiar with the error: It’s the fallacy of assuming that because event A preceded event B,
therefore event A must have caused event B. Not necessarily!—the events may not be
causally linked at all. Chronology, as they say, does not necessarily mean causality. In the
same way that the stimulus cites the rain as a cause of the hurricanes simply because the
rain came first, so (C) argues that playing competitive sports must causally relate to one’s
entrepreneurial skills, simply because so many entrepreneurs played sports first. Note that
the concept of causality is totally absent from the other four choices, which wander far
afield.

(A) assumes that most people who sleep well are healthy—a dubious assumption, but not
the flaw that the stimulus demonstrates.

(B) is focused on a comparison between long and short city blocks. No such comparison can
be found in the stimulus.

(D) argues opposites: Since sun opens the blossoms, dark must close them. Dubious, sure,
but the stimulus features no such opposition.

(E)’s conclusion is a prediction; and it reasons from a general statement about events to a
particular claim about liberalization. The stimulus features neither of these tactics.

• The “algebraic” approach to Parallel Logic won’t always pay off, especially when the
argument gets complex. Get in the habit of characterizing prose in general terms.
When you characterize the conclusion in (E), for instance, as a prediction, it’s easy to
discard (E) promptly, because the stimulus has nothing to do with prediction
whatsoever.

© K A PL A N 73
LSAT PREP ________________________________________________________________ LSAT Test I Explained: Section IV

11. (B)
The author believes that it is more important to criticize democracies that have violated
human rights than it is to criticize dictatorships that have committed more violent offenses.
Why? Among other claims, the author indicates that the violations committed by
democracies are worse (“more reprehensible”) than those committed by dictators, since the
actions of a democracy reflect the will of the people. So the violations committed by
democracies can be more reprehensible even if they are less violent. Then, as choice (B) has
it, it must be possible that some human rights violations can be worse than other human
rights violations that are more violent.

(A) This choice should read “All governments commit some inexcusable and
reprehensible acts.” (Emphasis added.) The version of this test that came with the LSDAS
Registration and Information book may contain an error, substituting “same” for “some,”
but in either case, (A) is not a valid inference. For all we know, at least one government is
squeaky-clean.

(C) Even though the author claims that criticism would have more of an effect on a
democracy, that’s no basis for claiming that criticism would have no effect on a
dictatorship.

(D) is out of the scope. The argument never addresses the relative frequency of human
rights violations.

(E) is an irrelevant distinction. The stimulus never mentions those that merely claim to
represent the will of the people.

• Once again (see Q. 9), the correct answer to an inference question need not relate to
every point made in the stimulus, but it must be necessarily true.

• Errors on the LSAT have been known to happen, but they are extremely rare. On
your test, if you think you see a problem with the question, you’ve probably missed
something, especially in Logic Games. In any case, don’t spend too much time on
one question. When you are really stuck, make an educated guess and move on.

12. (A)
This stimulus presents a classic correlation-causation argument. A study found that
smokers are more likely to snore than nonsmokers, even though snoring is not particularly
common in either group. From this, the author of the study concludes that smoking alone
can cause snoring. Not so fast. All we know is that two conditions (smoking and snoring)
are connected, and that isn’t enough to conclude that the first condition (smoking) causes
the second (snoring). For all we know, the causal relationship could be the other way
around—snoring could cause smoking. Although that seems somewhat unlikely in this
case, nothing rules this possibility out. Alternatively, both conditions could be caused by
some third factor (such as stress), as (A) suggests. Thus (A), if true, would explain how
smoking and snoring are correlated without implying a causal relationship between them.
So (A) weakens the argument by providing an alternative explanation of the author’s
evidence.

74 © K A PL A N
LSAT PREP ________________________________________________________________ LSAT Test I Explained: Section IV

(B) provides evidence concerning the causes of smoking, but this argument concerns the
effects of smoking, and whether one of them is snoring. By ignoring smoking’s effects in
general, and the issue of snoring in particular, (B) misses the point.

(C) and (D) both address the degree of correlation between smoking and snoring, whereas
the author only addresses the question of whether any causal relationship exists. So (C) and
(D) might be relevant to the question of the extent of the connection between smoking and
snoring, but they are irrelevant to this argument, which only claims that smoking can lead
to snoring, at least some of the time. Furthermore, choice (D) is practically inferable from
the evidence stating that snoring is not common among either smokers or non-smokers.

(E) has it backwards. Whereas the credited response suggests that a third condition could
cause both smoking and snoring, (E) suggests that both smoking and snoring cause a third
condition. This has nothing to do with the author’s conclusion that smoking can cause
snoring, so it in no way weakens the argument.

• The concepts of correlation and causation are long-time favorites on the LSAT, and
this question is a prime example of how they are tested. Remember, the fact that two
conditions (like smoking and snoring) tend to happen together does not show that
one causes the other. The causal relationship might be the other way around, or there
might be no causal relationship between them at all.

• Every question is worth one point, but some questions take less time to answer. Be
sure to get to them, no matter where they appear in the section. Question 12, for
instance, has a short stimulus and short answer choices, and could certainly have
been answered in less time than it would take to answer a typical Parallel Reasoning
question.

13. (B)
Will Rogers said “all politics is applesauce,” and this author might agree. The voters are
not getting the information they need to make good decisions, since they never learn about
the substantive positions that the candidates take on important issues. Instead, all they get
is reports on the moves and countermoves of the campaign advisors. Therefore, the
advisors should get out of the way, so that the press can report on the candidates’ positions
on substantive issues. What does that assume? For one, it assumes that the candidates have
positions on substantive policy issues, (B). If they have no positions on substantive policy
issues, then getting rid of the campaign advisors won’t help; the voters still won’t know the
candidates’ positions on the issues, since there are no positions upon which a report can be
based.

(A) Chess is mentioned merely to illustrate the author’s point; certainly, chess need not be
the most appropriate analogy in this case in order for the argument to stand.

(C) is irrelevant. The argument does not address the issues of how the substantive issues in
the campaign are determined. Instead, it concerns how the candidates’ positions on those
issues can be discerned, however they came about.

© K A PL A N 75
LSAT PREP ________________________________________________________________ LSAT Test I Explained: Section IV

(D) would weaken the argument. If the voters don’t care enough about the election to pay
attention, then it wouldn’t matter whether the campaign advisors stayed out of the
limelight.

(E) conflicts with the stimulus, which indicates that the press is reporting on the political
process and ignoring substantive issues. If the press can attend to one and ignore the other,
then they must be different, and so (E) is inconsistent with the evidence.

• Many LSAT stimuli present two alternatives, but discuss only one of them.
Whenever that happens, the central assumption in the argument will likely depend
upon the characteristics of the other, unmentioned alternative. For example, in this
question the author explains that reports on the activities of campaign advisors is
not providing enough information to the voters, and argues that those advisors
should stay out of the way so that press coverage can shift to the candidates
themselves. But we don’t know anything about the candidates yet, and so we have to
assume something about them if the author’s argument is to hold water.

• Use the Kaplan Denial Test to confirm that a choice is a necessary assumption. If the
negation of a choice will defeat the argument, then that choice must be necessary to
the argument. On the other hand, if a choice can be false without damaging the
argument, then that choice cannot be an assumption.

14. (E)
Next we’re asked to find a valid inference. In a complicated stimulus like this one, it can be
difficult to pre-phrase an answer to an inference question, so there’s nothing to do but
attack the choices. If the press reports only on campaign advisors, and the voters do not
currently have enough information to make informed decisions, then those reports are not
providing all of the information that the voters need to make those decisions, (E). If
reporting on the campaign advisors alone was doing the job, then the voters wouldn’t be in
the “information-deprived” situation described in the stimulus.

(A), (C), and (D) are out of the scope. The preferences of the candidates with respect to
media coverage, (A), their beliefs about the political process, (C), and the relative ease of
different subjects of reporting, (D), are all issues that are never mentioned.

(B) has a few problems. First, it is the campaign advisors who are in the limelight, not the
press. Second, similar to the other wrong choices mentioned above, we’re not given
enough information to infer anything about what the press enjoys.

• Focus on the scope of the argument, and eliminate choices that depart from it. Stick
to the argument that you are given, and don’t be distracted by irrelevant issues
brought up in the answer choices.

• Sometimes an answer choice will make a claim about someone’s intention or beliefs,
even though the stimulus describes only their behavior. In this question, choices
(A), (B), and (C) all commit this scope shift. These choices can be tempting when the
claim about the intention or belief could be true (or is often true in the real world).

76 © K A PL A N
LSAT PREP ________________________________________________________________ LSAT Test I Explained: Section IV

Read critically and don’t infer anything about beliefs or attitudes without concrete
support from the text.

• The Kaplan Denial Test can be used to check the choices in Inference questions, too!
Remember that a valid inference must be true given the information in the stimulus.
So any choice that could be false cannot be a valid inference. Here, if “reporting on
campaign advisors” was providing enough info for the voters to make informed
decisions, the argument takes a nose dive.

15. (E)
Most humans are right-handed, but what about animals? Are animals equally likely to
have a preference for their left limbs as their right limbs? Certain studies say that animals
will be equally likely to be “right-handed” as “left-handed,” but the author disagrees. In
support, the author notes that dogs often “shake hands” with their right paw. OK, so what?
Suppose, as (E) suggests, that dogs are influenced by their masters in learning to perform
tricks like “shaking hands.” In that case, we would expect that since most masters
(presumably humans) are right-handed, most dogs would be taught to perform tricks in a
“right-pawed” manner. So if (E) is true, it isn’t the dogs that are showing a preference for
their right limbs at all. Rather, dogs that “shake hands” with their right paws are merely
reflecting the preferences of their masters.

(A) and (D) are out of the scope. We’re asked to explain why the “shaking hands” evidence
does not weaken the claim that animals will be equally likely to favor their right limbs as
opposed to their left limbs. Answer choices (A) and (D) completely ignore the curious
findings regarding “shaking hands,” and so cannot help in a defense against this supposed
counterexample. Choices (A) and (D) do provide some evidence that dogs show no
preference for their right limbs in other respects, which might help counter the claim that
dogs have a tendency to be “right-handed” in all respects, but that’s the subject of another
question.

(B) The evidence is unrepresentative, but not for the reason in (B). There’s no grounds for
being especially suspicious of “front-paw-only” behavior.

(C) implies that dogs are exempt from the kinds of societal pressures and inconveniences
facing left-handed humans. Inferably, then, dogs aren’t pressured in any way to shake
hands with their right paw—they simply do so naturally. Thus, if anything, (C) strengthens
the validity of the counterexample by ruling out an alternative explanation for the dogs’
noted behavior. (Notice how this differs from correct choice (E), which provides an
alternative explanation for why dogs usually “shake hands” with their right paw, and thus
damages the validity of the counterexample.)

• Whenever a survey is used as evidence, think “unrepresentative sample.” Look for


something about the evidence that suggests bias or unreliability.

• Pre-phrase whenever you can. You’ll save time, and you’ll be less likely to fall for
distracting answer choices. Why spend time figuring out why the four wrong
answers are wrong when it’s easier and faster to recognize the one that’s right?

© K A PL A N 77
LSAT PREP ________________________________________________________________ LSAT Test I Explained: Section IV

• Always read the question stem first! You’ll get valuable information about your task,
and you may even get a preview of the topic and scope of the argument.

16. (A)
Reading the question stem first pays off big here. We’re looking for what the student
(mistakenly) thought the professor said. The student claims to have a counterexample to the
professor’s “genetic law.” She has Type O blood, even though her father does not have
Type O blood. What claim would this counter? It would certainly show that it is not
necessary for a Type O child to have a Type O parent. So choice (A), claiming that only
people with Type O blood can have children with Type O blood, would be disproved by
the student’s rejoinder. For this reason, (A) must represent the student’s misinterpretation
of the professor’s remark.

(B) If (B) were the answer, the student would have to have said something like “My father
has Type O blood and I have Type B blood.”

(C) is consistent with the student’s claim, so (C) could not be the claim to which the student
objects.

(D), (E) What other children? What other parent? The student never mentions either of
them, so there’s no basis for concluding that the student is thinking of either one.

• Always read the question stem first. Sometimes, you get a sneak preview of the
argument’s topic and scope, and you always get information to direct your attack on
the stimulus.

• When the second speaker has misinterpreted the first speaker’s statement, focus on
the second speaker’s position. Think about the claim that the second speaker must
be countering; since there’s been a misinterpretation, expect that this claim won’t
mesh with the first speaker’s actual statement.

17. (B)
Here we get another curious situation: R-Bar Beans sell fewer cans nationally than its
competitors, even though R-Bar Beans are preferred among all age groups in a recent
survey. This seems odd, because we would normally expect that people would buy more
of a product that they find superior. So what’s going on? Four of the choices, if true, would
help resolve the discrepancy, and one of them (the credited response) would not. If (B)
were true, and the preference for R-Bar Beans is more pronounced in some groups as
opposed to others, then we would expect R-Bar Beans to have more success in those R-Bar
favorite groups. However, we would still expect R-Bar Beans to have the overall lead in
bean sales, since R-Bar is preferred among all age groups. That isn’t the case, as R-Bar ranks
at best third in sales. (B) adds nothing to our understanding of how R-Bar compares with
the other brands, and thus doesn’t help us solve the mystery.

(A) If the other beans are cheaper than R-Bar Beans, then it’s not surprising that some
people buy them even if they think R-Bar Beans taste better. It’s no shock that Hondas
outsell Rolls Royces, even though most people would prefer to drive a Rolls.

78 © K A PL A N
LSAT PREP ________________________________________________________________ LSAT Test I Explained: Section IV

(C) If the survey is not representative of the national market, and if R-Bar Beans are not
even available nationwide, then it’s no wonder that R-Bar isn’t #1.

(D) If many stores refuse to carry R-Bar, then reduced sales for R-Bar would be expected.

(E) solves the mystery by telling us that R-Bar sales figures reflect only three months of
sales as compared with a full year of sales for the two competitors. No wonder R-Bar sold
far fewer cans of beans last year than the others even though the recent survey shows that
customers prefer R-Bar. Once R-Bar does catch on, though, there’s a good chance that Texas
T and Aunt Sally’s will be “has-beans.”

• The first step in Paradox questions is identifying the apparent paradox. Once you
understand why the result was unexpected, you should have a strong grip on the
scope of the argument, which will help you determine what will resolve the apparent
paradox.

• Accept the evidence that you’re given, but be skeptical about the gap between the
evidence and the conclusion. Any issue ignored by the author is fair game.

• “Each of the following EXCEPT” questions can be very time-consuming. If you’re


short on time, you may want to tackle other questions first.

18. (E)
Some cosmetics firms claim that the new product-safety tests, which use cultures of human
cells, reduce the need to conduct tests on live animals. Why would the firms devote
resources to perfecting the tests? They must attach some value to reducing the scope of
live-animal tests, all other things being equal. Therefore, the managers of those firms, the
ones making the decisions, must believe that as long as safety measures can be maintained,
it is better not to perform tests on live animals, choice (E).

(A) has two problems. First off, “pressure” is too strong a word to fit the scope of this
stimulus. All we know is that the firms are committed to the concept of the new product-
safety tests that will reduce the need for testing animals. We can’t rightfully infer from this
that the firms are being pressured into this commitment. But even if we do infer this outside
pressure, there’s still no way to conclude who initiated such pressure.

(B) Consumers and their preferences are one step removed from the information in the
stimulus. There’s no way for us to conclude which products consumers are more likely to
buy.

(C) Just like “consumer preferences” in (B) above, financial consultants’ beliefs regarding
the costs associated with the new product-safety tests are outside the scope of the
argument.

(D) It is impossible for us to determine the number of product tests necessary using the
new system as opposed to the old. We simply aren’t given enough information to validate
the researchers’ beliefs stated in (D).

© K A PL A N 79
LSAT PREP ________________________________________________________________ LSAT Test I Explained: Section IV

• Don’t let your outside knowledge interfere with your handling of the questions.
Sometimes, a question, stimulus, or passage will touch on an issue about which you
have a personal opinion. Always stick to the facts as they appear on the page.

• Pay strict attention to the scope of the passage, and you should be able to
differentiate between what you know and what you don’t know from any given
stimulus. Here, (A) through (D) discuss issues that are at least one step removed
from the thrust of the short passage. Therefore, there’s no way any of these choices
can serve as a valid conclusion.

19. (A)
When the testmakers give you a question that can be decoded quickly with Keywords and
structural clues, thank them, get the right answer, and move on. We’re asked for the main
point of the passage, and so you’ll want to key in whenever the author expresses her
opinion. The first sentence merely asks if there is a necessary relationship between negative
feelings toward the elderly and plucking out gray hairs. The second sentence begins with
the Keyword “unless,” signaling an exception, followed by the strong statement of opinion
“there is no necessary connection.” The third sentence makes a distinction between
attitudes that supports the notion that there is no necessary connection. The fourth sentence
begins with the Keyword “furthermore,” signaling additional supporting evidence, and
the fifth sentence begins with the Keyword “but,” signaling a contrast to the supporting
evidence in the previous sentence. Neither of these sentences can be the main point. So
where are the expressions of opinion? The second sentence is the only qualifier. It claims
that it would be premature to conclude that there is a necessary connection between hair-
plucking and having negative attitudes about the elderly, which is paraphrased by choice
(A).

(B) is a statement with which the author would agree, but the differing evaluations of these
attitudes is used to support the author’s argument that there is no necessary connection
between them. (B) is a step along the way, but not the final destination.

(C) The two claims mentioned in (C) are not linked to each other by the passage. They are
both used to support a further point, the view that there is no necessary connection
between disliking aging and disliking the elderly.

(D) brings in a new term, “fine,” without support from the stimulus. Furthermore, “being
elderly” is not part of the distinction drawn in the stimulus—that distinction is between
people’s attitude toward the elderly and their feelings about the process of getting old
themselves.

(E), like (B), is a view with which the author would agree, but as discussed above, the
author’s moral arguments are used to support the main point that there is no necessary
connection between these two attitudes.

• Use Keywords and structural signals to help you navigate through Logical
Reasoning stimuli and Reading Comprehension passages. To find the main point,
watch out whenever the author expresses an opinion, and pay less attention to
supporting evidence.

80 © K A PL A N
LSAT PREP ________________________________________________________________ LSAT Test I Explained: Section IV

• Many Main Point answer choices mention elements that were present in the
stimulus, but were only a step along the way to the author’s real conclusion. So don’t
just choose the first choice that contains something you recognize. Ask yourself if
that information was itself used as evidence to bolster another larger point.

20. (D)
Next we’re asked to find the one method of argument out of five the author does not use.
While the author does criticize those who hold negative attitudes about the elderly, the
author does not “discredit a common stereotype about the elderly,” choice (D). The
author does imply that aging is associated with impaired eyesight and hearing, but far
from discrediting this information, the author uses it in showing why disliking those
features may not be associated with disliking the elderly.

(A) In the first sentence, the author points to the lack of research to support the assertion
that plucking gray hairs and harboring negative attitudes toward the elderly are
connected.

(B) In pointing to a lack of evidence, applying general principles, and in making a


distinction, the author could fairly be described as appealing to reason. Further, it’s hard
to imagine what this stimulus would have to look like for (B) to be the right answer. Only a
very sketchy (perhaps even circular) argument doesn’t appeal to reason in some way or
another.

(C) In the fourth sentence, the author cites the general principle that having negative
attitudes toward the old because they are old is immoral.

(E) In the third sentence, disliking the elderly is distinguished from disliking conditions
associated with the aging process.

• While working on Method of Argument questions, get to know the “usual


suspects,” the choices that appear in many questions. Pointing to the lack of
evidence on the other side (A), appealing to general principles (C), and making
distinctions (E), are common argumentative techniques on the LSAT.

21. (D)
The valid inference we’re asked to find is merely the contrapositive of one of the statements
in the fourth sentence. The author states that “a society that has no laws has no crimes.” In
“if-then” format, this means “if a society has no laws, then that society has no crimes.” To
form the contrapositive of any if-then statement, reverse the terms and negate (deny) both.
To deny that a society has no laws is to claim that the society has at least some laws, and to
deny that a society has no crimes is to claim that the society has at least some crimes. So
reversing the terms and negating both leaves us with “a society that has some crimes has
some laws,” which is choice (D).

(A), (B) No, crimes guarantee laws, not the other way around. For all we know, at least one
lawful society is crime-free. (Did you notice that these two choices are contrapositives of
each other?)

© K A PL A N 81
LSAT PREP ________________________________________________________________ LSAT Test I Explained: Section IV

(C) and (E) introduce a new term, “many,” without support from the stimulus. For all we
know, a society could have many laws without any crimes at all (C), and it is also possible
that a crime-plagued society has few laws, (all the criminals could be breaking the same
law, for instance), which axes (E).

• Know your contrapositives! You’ll need them in Logic Games, and in Logical
Reasoning as well. Many correct answers to inference questions are based on the
contrapositive of if-then statements.

• Whenever you are given a statement in if-then form, you may safely infer the
contrapositive of that statement. To form the contrapositive of an if-then statement,
reverse the terms and negate both.

22. (B)
Is there a connection between the amount of rain in the Galapagos and the size of finch
species that tend to thrive there? Finches eat seeds, and the amount of rain helps to
determine the kind of seeds that are present. In times of little rain, large finch species have
the upper hand, since only they have bills large enough to crack the large, hard seeds.
However, rainy times favor plants with small seeds, which means the small finch species
have plenty to eat, as their inability to crack the large, hard seeds is no longer so significant.
However, the large finches cannot eat fast enough to meet their extraordinary energy
requirements. Why would that be? The author must be assuming (B), that during rainy
weather, fewer large, hard seeds are being produced. After all, if the supply of large, hard
seeds were as high as ever in rainy times, then the large species should be able to thrive,
since that same level of large, hard seeds was enough to allow the large species to thrive in
times of less rain. So if rain really does play a role in which finch species thrive, then the
author must be assuming (B).

(A) mixes up terms. The author describes large, hard seeds, and small seeds, but there’s no
telling what difference small, hard seeds could make.

(C) Individual finches could stay about the same size during rainy periods without
injuring the argument. If they did grow to big-bird size during those periods, they would
encounter the same problems as the larger species, but there’s no reason to believe that they
would encounter such a growth spurt.

(D) is irrelevant. The argument concerns what happens when the climate changes, and not
the relative frequency of different climates.

(E) For all we know, all species have the ability to crack the small seeds, so there’s no need
to assume that the small seeds can be digested without cracking.

• Read the question stem first. Sometimes, as in this question, you are told the topic
and even the scope of the argument. It pays to know this information before you
tackle the stimulus.

82 © K A PL A N
LSAT PREP ________________________________________________________________ LSAT Test I Explained: Section IV

• Don’t “fill in the blanks” for the author. Anything necessary to the conclusion that is
left out of the argument must be an assumption. Sometimes, an assumption is so
basic that it is easy to overlook, but this kind of assumption is often the key to the
right answer.

23. (C)
We’re looking for a situation that would favor Fring’s view over Blatt’s view. Blatt thinks
the opinions offered by consultants are worth the money. Fring believes that consultants
are hired so that executives will have someone to blame when things go wrong, and (here’s
the key phrase) executives prefer to pay more for consultants, since increased pay means
increased responsibility. If (C) were true, and a reduction in consulting fees led to a
reduction in business, then Fring’s claim that executives prefer to be over-charged would
be strengthened.

(A), (B) The issue is the reason for hiring a consultant. Without knowing the cost of a
consultant or the value associated with his or her advice, the decision to hire or not hire a
consultant does not favor either speaker.

(D) would strengthen Blatt’s position, since a link between consultant fees and money
saved by the company suggests that consultants have some worth after all.

(E) We can’t tell if the money paid to the consultant was worth it (since the stores are
marginally profitable at first), and we don’t know whether the company hired the
consultant to help make decisions or just to have someone to accept any future blame, so
it’s hard to see how (E) helps either side.

• Get a strong grip on the scope of the argument before you hit the choices. If you
know the real point at issue, then you’ll zero in on the choice that impacts on that
point in the proper way.

© K A PL A N 83
LSAT PREP ________________________________________________________________ LSAT Test I Explained: Section IV

24. (E)
Here’s another Parallel Logic example readily reduced to algebra: All X (contemporary
ads) are Y (efforts to persuade). Some X (some contemporary ads) are Z (morally
reprehensible). Therefore some Y (efforts to persuade) are Z (morally reprehensible). In
correct choice (E), sonnets = X, short poems = Y, and thematically pluralistic poems
(whatever that means) = Z.

(A) contains only two pieces of evidence, not the requisite three; and (A)’s “No X are Y”
evidence has no parallel in the stimulus.

(B) and (D), like (A), fail to provide the requisite three pieces of evidence. Moreover, (B)’s
conclusion is about “reliability,” a concept that is never mentioned in the evidence (notice
that in both the stimulus and correct choice (E), both terms in the conclusion do appear in
the evidence). And (D)’s conclusion is that “All X are Y”—very different from the stimulus.

(C) All X (good managers) are Y (make important decisions), while some Z (managers in
general) are not Y (fail to make important decisions). Therefore some Z are not X. Way far
afield from the stimulus.

• In Parallel Logic, watch for the correct answer to rearrange (as (E) does here) the
order in which the evidence and conclusion are presented. The placement of the
pieces is a stylistic decision, not a logical one. As long as (E) has a “Some X are Z”
conclusion—which it does—then its conclusion is parallel to that of the stimulus,
irrespective of where in the paragraph the conclusion appears. Ditto for the
evidence: The logic is the same even if the premises are presented in a different
order.

84 © K A PL A N
I.N. LL3122 Rev.A Printed in the USA

You might also like